ORTHOPEDIC MCQS OB 20 BASIC 2

ORTHOPEDIC MCQS OB 20 BASIC 2

  1.  

  1. Which of the following exhibits osteoinductive and osteoconductive properties when treating the injury shown in Figure A?

 

 

 

Corrent answer: 2

 

 

 

 

Neutrophils are the first cells to appear following acute muscle injury.

 

 

 

 

Tidball showed that mononucleated cells that normally reside in muscle are activated resulting in chemotactic signaling to inflammatory cells. These signaling molecules cause a massive influx of neutrophils to move to the injury site releasing inflammatory cytokines, resulting in swelling. Following this

 

 

initial phase is an increase in macrophages that phagocytose debris. Finally, there is an increase in a second subpopulation of macrophages associated with muscle regeneration

and scarring. Neutrophils and their generation of free radicals is thought to be associated with increased scarring and muscle damage.

 

 

Figure A demonstrates a quadriceps contusion and these are treated with icing immobilization in 120 degrees of knee flexion for 24 hours followed by

 

 

therapy.

 

 


Osteoblasts Neutrophils Myoblasts Lymphoblasts Eosinophils Which of the following cells appear first at the site of injury?

 

 

 

 

 

 

 

A collegiate football player is hit in the thigh by an opposing player's helmet. Radiographs are unremarkable and a clinical image of the injury is shown in Figure

 

 

 

 

 

 

 

Corrent answer: 2

 

 

 

 

The above description is termed a type II error (answer 2). To put it succinctly, this is a false negative (accepting the null hypothesis when there is actually a true difference). Type II errors can be minimized by making sure that a study is properly powered via a power analysis so that you can detect true differences if they do exist.

 

 

 

 

Lochner et al. studied the rates of type II error in 117 randomized controlled trials involving fracture care. They found that the type-II error rates in these trials exceeded accepted standards and suggest that the future authors could reduce these rates with pre-study power and sample-size calculations.

 

 

Incorrect Answers:

1. Type I error-false positive error, meaning that a study finds groups to be diffferent when they actually are not. Put another way, the null hypothesis is incorrectly rejected.

Bias-there are many types, however in general can be defined as a systemic inaccuracy in data due to the characteristics of the process employed in the creation, collection, manipulation, and presentation of data, or due to faulty sample design of the estimating technique. Negative predictive value-The proportion of patients with a negative test result who actually do not have the disease/condition of interest. Positive predictive value-The proportion of patients with a positive test result that actually have the disease/condition of interest. Type I error Type II error Bias Negative predictive value Positive predictive value

 

 

Corrent answer: 1

 

 

 

 

Warfarin inhibits the synthesis of vitamin K-dependent coagulation factors II, VII, IX, and X and anticoagulant proteins C and S.

 

 

 

 

Warfarin inhibits the C1 subunit of the vitamin K 2,3-epoxide reductase (VKORC1) enzyme. This results in depletion of the reduced form of vitamin K (vitamin KH2) and limits the gamma-carboxylation of the vitamin K-dependent coagulant proteins. Warfarin requires monitoring of INR to ensure therapeutic dosing (target level INR 2-3). Reversal of warfarin may be achieved with vitamin K (takes up to 3 days) or fresh frozen plasma (acts immediately).

 

 

 

 

Leiberman, et al reviewed the new evidence-based guidelines for venous thromboembolicprophylaxis after total joint arthroplasty developed by the American Academy of Orthopedic Surgeons (AAOS) and the American College

 

 

of Chest Physicians (AACP). The AAOS was unable to make a recommendation with respect to the selection of specific prophylaxis regimen or duration. The ACCP recommended one of the following for a minimum of 14-days: warfarin, LMWH, fondaparinux, aspirin, rivaroxiban, dabigatran, apixaban, or portable mechanical compression.

 

 

Einhorn et al review the pathophysiology and treatment of venous thromboembolic disease in orthopaedic surgery. The authors note that a large investigator-initiated, independently randomized clinical study is warranted for objectively assessing the safety and efficacy of available thromboprophylaxis regimens.

 

 

 

 

Illustration A demonstrates the targets for anticoagulation on the coagulation cascade.

 

 

Incorrect Answers:

Answer 2: Heparin binds and enhances the ability of antithrombin III to inhibit factors IIa, III, and Xa.

Answer 3: Aspirin inhibits the production of prostaglandins and thromboxanes through irreversible inactivation of the cyclooxygenase enzyme.

Answer 4: Clopidogrel acts by inhibiting the ADP receptor on platelet cell membranes, which is important in activation of platelets and eventual cross- linking.

Answer 5: Enoxaparin enhances the ability of antithrombin III to inhibit factors IIa (thrombin) and Xa.

 


 

 

 

 

 

 

 

 

What term in statistics defines accepting the null hypothesis when it is in fact not true?

 


Warfarin Heparin Aspirin Clopidogrel Enoxaparin

 

 

 

 

 

 

 

 

The key laboratory factor that differentiates nutritional from hypophosphatemic rickets is an elevated PTH. Tortolani et al. review the

differential diagnosis of bone mineral density deficiency in their JAAOS review. As discussed, nutritional rickets shows low to normal serum calcium, low serum phosphate, elevated alkaline phosphatase, and elevated parathyroid hormone. While nutritional rickets is due to dietary deficiency, hypophosphatemic rickets is caused by the inability of kidney proximal tubules to reabsorb phosphate due to mutated PHEX gene, found on the X chromosome. PHEX is thought to protect extracellular matrix glycoproteins from

proteolysis. Hypophosphatemic rickets shares many clinical manifestations with nutritional rickets, but shows PTH levels that are not elevated, even with calcium and phosphate abnormalities (pseudohypoparathyroidism). Loeffler and Sherman evaluated the addition of phosphate to the standard vitamin D therapy in hypophosphatemic rickets on

long-term growth and deformity correction. They found no additional benefit of this therapy and as such, it is currently not recommended. Ferris et al. review their treatment of 19 patients with hypophosphatemic rickets and found that staged surgical intervention afforded better results but these patients, even with intervention, developed early degenerative joint disease.

 

 

 

 

 

Which drug works by preventing gamma carboxylation in factor X, factor IX, and prothrombin?

 

 

 

 

 




Low serum phosphate, elevated alkaline phosphatase, elevated PTH Low serum phosphate, elevated alkaline phosphatase, normal PTH Low serum phosphate, elevated alkaline phosphatase, decreased PTH Elevated serum phosphate, elevated alkaline phosphatase, elevated PTH Elevated serum phosphate, decreased alkaline phosphatase, decreased PTH Corrent answer: 1

 

 

Corrent answer: 5

 

 

 

 

Usage of low-molecular-weight heparins (LMWH) have been shown to have an increased rate of postoperative hematomas and wound complications in several large studies.

 

 

 

 

The referenced study by Dorr et al noted that wound complications and hematomas only existed in warfarin or low-molecular weight heparin patients, and not in patients treated with ASA, clopidogrel, or compression devices.

 

 

The referenced study by Lee et al reviews the complication of retroperitoneal hematoma during LWWH usage.

 

 

 

 

 

You are seeing a 4-year-old girl for leg deformities on a mission trip to Haiti. Clinical photograph and radiographs of her lower extremities and wrist are shown in Figures A-C. What laboratory studies would help confirm a nutritional deficiency as opposed to an X- linked genetic disorder as a cause of her condition?


Pneumonia Fatal pulmonary embolism Inferior vena cava filter placement Renal failure Postoperative hematoma

 

 

 

 

 

 

 

Corrent answer: 3

 

 

 

 

A material undergoing plastic deformation will not return to its original form once the stress is removed. This occurs once a material has been subject to stress past the yield strength, also called the yield point. Prior to the yield point, elastic deformation occurs, and the material will return to its original form once the stress is removed. Illustration A demonstrates these definitions in the classic stress-strain curve. The amount of energy a material can absorb

 

 

before failure is defined as toughness. Ultimate strength is the highest point on the stress- strain curve. It represents the maximum stress a material can absorb while being stretched before "necking", when the cross-sectional area of the material begins to contract. Fatigue strength refers to cyclic testing of a material. Also called fatigue limit or endurance limit, it is the amount of cyclic

stress that can be applied to a material before failure. Endurance limit has also been used to define the maximum level of stress that can be applied to a

material cyclically and never cause failure.

 

 

 

 

 

 

 

Low-molecular-weight heparin has been shown to have an increased rate of which of the following when compared to aspirin, clopidogrel, and compression devices?


Toughness Ultimate strength Yield strength Fatigue strength Endurance limit

 

 

 

 

 

 

 

 

 

Corrent answer: 5

 

 

 

 

Figure E shows an infraisthmal femur fracture for which a intramedullary nail remains the gold standard treatment option.

 

 

 

 

Fractures that have historically been treated successfully with traditional non- locking plates (eg, humeral shaft, both-bone forearm, lateral malleolus)

 

 

require locked fixation in osteoporotic bone or fractures with segmental loss or short-end segments as a result of comminution. Fractures that have demonstrated an increased risk of mechanical failure with unlocked plates include proximal humerus, distal radius, distal femur, and proximal tibia.

 

 

Haidukewych et al, in their review article, state that locked plates may prove to be ideal for: 1) indirect fracture reduction, as they can tolerate imperfect reduction and need not be placed on the tension side of the bone; 2) diaphyseal/metaphyseal fractures in osteoporotic bone; 3) the bridging of severely comminuted fractures to minimize soft tissue damage; and 4) the plating of fractures where, due to anatomic constraints, a compression plate may not be placed on the tension side of the fracture.

 

 

Incorrect Answers:

Answer 1: Proximal humerus fractures are commonly treated with proximal humeral locking plates.

Answer 2: Periprosthetic distal femur fractures are typically treated with locking or hybrid technology due to osteopenia and short-segment fixation. Answer 3: A periprosthetic femur fracture (THA) is often associated with a short segment proximally that requires use of cables and locking screws (unicortical or bicortical), and distal fixation may be hybrid in nature.

Answer 4: Distal radius fractures are often treated with locking technology due to osteopenia and fixed angle support of the smaller distal fragment(s).

 

 

 

 

 

Which of the following defines the stress at which a material begins to undergo plastic deformation?

 

 

 

 

 


 

 


Figure A Figure B Figure C Figure D Figure E

 

 

 

 

Corrent answer: 5

 

 

 

 

Fasciotomy related lawsuits have the highest rate of successful malpractice claims (48% success rate), followed by lawsuits related to musculoskeletal infections (43%).

 

 

 

 

A previous study of malpractice claims on orthopaedic surgeons reveals that osteoarthritis was the most common diagnosis associated with these suits (21% of all suits). However, the most recent data (2019) reveals that fracture related suits are more common than arthroplasty malpractice suits (2nd most common). The trend of increased arthroplasty related lawsuits over the last decade may reflect the aging of the U.S. population, as baby boomers enter their sixth decade of life.

 

 

 

 

A study of malpractice claims filed against orthopaedic surgeons from 1985-

 

 

1998 that were resolved by Physician Insurers Association of America (PIAA) found that femur fracture ranked first on the list of most frequent suits by diagnosis. Technical problems in femur fracture care were five times more likely to be responsible for the suit than the other most common causes: failure to diagnose, infection, or death.

 

 

 

 

 

Which of the following fracture patterns (Figures A through E) has the least amount of evidence-based support for use of locking or hybrid plating techniques?


Total hip arthroplasty Femur fracture Meniscal tear Herniated intervertebral disc Compartment syndrome

 

 

 

 

 

 

 

Corrent answer: 3

 

 

 

 

Osteomyelitis is defined as the acute/chronic inflammatory process in the bone secondary to infection with pyogenic organisms. There are two described methods of infection: direct innoculation or hematogenous spread of organisms. Puncture wounds through athletic shoes are one example of direct innoculation causing osteomyelitis, most commonly Pseudomonas species.

 

 

 

 

Hematogenous osteomyelitis caused by Salmonella species is characteristic of patients

 

 

with sickle cell disease. Salmonella bacteria is able to infect patients with sickle cell disease due to micro-infarcts in their circulatory system, specifically in the intestines. Once in the blood, sickle cell disease patients have a decreased ability to fight potential infections due to their functional asplenia and impaired complement activity.

 

 

Newborns, younger than 4 mo, are typically affected by Staphylococcus aureus, Enterobacter species, and group A and B Streptococcus species. Intravenous drug abusers and dialysis patients who develop osteomyelitis are often infected by Staphylococcus, Enterobacter, Pseudomonas, or Streptoccus species.

 

 

 

 

An orthopedic surgeon meets with an attorney because a malpractice claim has been filed against him. Which of the following diagnoses is most likely to result in a successful malpractice claim?


Neonates Intravenous drug abusers Patients with sickle cell disease Patients with chronic kidney failure requiring dialysis Patients with puncture wounds through athletic shoes

 

 

 

 

Corrent answer: 3

 

 

 

 

Signing out to physician extenders instead of a resident physician is not cited as a source of decreased continuity of patient care. Communication and transfers in care have been cited as sources of decreased continuity of care as a sequlae of the 80-hour resident physician work week implemented by the ACGME.

 

 

Friedlander conducted a survey of orthopaedic surgery residents and found that though residents are personally happier, they were concerned about the possible loss of continuity of care.

 

 

An editorial in the NEJM by Okie discusses the delicate balance between optimal patient care and national limits on resident duty hours.

 

 

 

 

The cohort study by Horwitz et al analyzed 503 patient "sign-outs" and found a 5% rate of communication inadequacies including omitting key information about recent or scheduled events, tasks to complete, and rationale for assigned tasks.

 

 

 

 

 

 

 

Hematogenous osteomyelitis caused by Salmonella is most common in which of the following patient populations?


Omitting the patients' current clinical condition Omitting guidance for events likely to occur on the next shift Signing out to physician extenders instead of a resident physician Rationale not given for plans provided to the covering physician Tasks not assigned to the covering physician for completion

 

 

 

 

 

 

 

Corrent answer: 2

 

 

 

 

Adjunctive therapies for bone healing are widely used and the mechanism of action is slowly being elucidated. Capacitive coupling involves externally placed electrodes with an alternating current which creates an electrical field. This stimulates calcium translocation which then activates calmodulin and upregulates many factors involved in bone healing (BMP, cyclic adenosine monophosphate (cAMP), and TGF-beta1).

 

 

 

 

Direct current(DC) stimulates an inflammatory-like response during fracture repair while

 

 

. Pulsed electromagnetic fields (PEMFs) cause calcification of fibrocartilage but not calcification of fibrous tissue.

 

 

Answer 1 describes the mechanism of action of direct current stimulation. Answer 4 describes ultrasound stimulation. Otter et al present a nice review of electromagnetic fields and their influence of fracture healing.

 

 

All of the following are cited as a concern for decreased continuity of care when "signing out" patient care from one provider to another EXCEPT?

Reduces oxygen concentration and increases local tissue pH Stimulates transmembrane calcium translocation via voltage-gated calcium channels Upregulates TNF alpha Transmits mechanical energy to stimulate bone formation Upregulates osteoclast activity

 

 

Corrent answer: 4

 

 

 

 

According to the Emergency Medical Treatment and Active Labor Act (EMTALA), transfer of a patient to a higher-level facility can be done with formal acceptance by the accepting facility. The benefits of transfer must outweigh the risks, and the exiting facility must not have the resources to properly handle that particular patient and/or situation.

 

 

 

 

According to the referenced article by Koval et al, the following had a greater odds ratio of being transferred to a level I facility: males, children, night transfers, Medicaid patients, and those with multiple comorbidities. Children and patients with multiple medical cormorbidities have the highest risk for hospital to hospital transfer.

 

 

 

 

 

 

 

What is the mechanism of action of capacitive coupling (CC) stimulation when used as an adjunctive therapy for bone healing?

Insurance level must be discussed with the accepting facility All appropriate laboratory workup must be completed The patient must be medically stable for transfer Exiting facility must not have resources to properly treat Cost(s) of transportation must be within reason

 

 

 

 

 

 

 

Corrent answer: 5

 

 

 

 

"Drop-off" refers to knee buckling or excessive knee flexion as the limb transitions from midstance (normally knee is extending) to terminal stance (the start of heel-off phase leads to knee flexion).

 

 

 

 

Knee flexion normally occurs along with the passing of the center of gravity over the metatarsophalangeal joints. If the body weight is carried over these joints prematurely, then early knee flexion or "drop-off" occurs.

 

 

 

 

Therefore, any prosthetic variable that decreases anterior support (i.e. dorsiflexed position of prosthetic foot, prosthetic foot too posterior, too flexible/soft of a keel, and a toe break that is placed too posterior) would accentuate this gait abnormality by moving the center of gravity more posterior. See Illustration A for details.

 

 

 

 

 

 

 

The Emergency Medical Treatment and Active Labor Act (EMTALA) requires which of the following?


Dorsiflex prosthetic foot Move foot more posterior Increase flexibility/softness of keel Move toe break of prosthesis more posterior Move foot more anterior

 

 

 

 

 

 

 

Corrent answer: 2

 

 

 

 

Bisphosphonates can be divided according to their chemical structure into the earlier non- nitrogen (e.g. clodronate and etidronate) and the newer nitrogen containing (e.g alendronate) groups. The Nitrogen containing group is significantly more potent. Nitrogen containing bisphosphonates target farnesyl diphosphate synthase, a component of the mevalonate (cholesterol) pathway. Non-nitrogen containing bisphosphonates result in cytotoxic ATP to accumulate with an end result of apoptosis. The JBJS article by Morris is a good overview

 

 

of bisphosphonate basic science, usages and adverse reactions.

A 35-year-old woman with a transtibial prosthesis is seen to have knee buckling (i.e "drop-off") during terminal stance. What prosthetic modification would correct this problem?


HMG-CoA reductase farnesyl diphosphate synthase tissue inhibitor of metalloprotease alkaline phosphatase calcitonin

 

 

Corrent answer: 2

 

 

 

 

For vertebrates, the axial skeleton derives from the somites, which are segmental units organized in pairs on both sides of the developing neural tube. Somites are formed in a rostro-to-caudal sequence by the epithelialization of mesenchymal cells at the rostral end of the presomitic mesoderm. The

 

 

function of the Hox genes is to regulate somitogenesis of the axial skeleton. It is generally accepted that a specific combination of Hox genes expressed at a particular somatic level determines the axial identity of the resulting structures.

 

 

Iimura et al & Turnpenny et al both provide articles which highlight the role that Hox genes and other regulatory genes play in controlling cellular movements during gastrulation, therefore contributing to body formation.

 

 

 

 

 

 

 

Nitrogen containing bisphosphonates inhibit osteoclasts by targeting what enzyme or hormone?


Create cross links between collagen fibers Regulate somitogenesis of the axial skeleton Stimulate calcification in the hypertrophic zone Cause a mutation which stimulates cell proliferation Mediate the activity of the polarizing zone

 

 

 

 

 

 

 

Corrent answer: 1

 

 

 

 

There are many modes of corrosion in orthopaedic implants and galvanic corrosion is a type of corrosion which results from an electrochemical potential created between two metals in a conductive medium.

 

 

Galvanic corrosion is often seen at the interface of metals (e.g plates and screws) when different metals are used. The conductive medium is usually serum or interstitial fluid.

 

 

Incorrect Answers:

 

 

Answer 2 describes fretting corrosion and answer 3 describes crevice corrosion. Answer 4 describes pitting corrosion and answer 5 does not make sense.

What is the function of the homeobox gene products in the axial skeleton?

 


Corrosion resulting from an electrochemical potential created between two metals in conductive medium Corrosion resulting from contact sites between materials under load Corrosion resulting from oxygen tension differences Corrosion from localized pits on metal surfaces Corrosion from allergic reaction

 

 

 

Corrent answer: 4

 

 

 

 

Demineralized bone matrix is obtained from allograft bone using a process of acid extraction which yields a product containing much of the collagen and non-collagenous protein components of bone including BMP.

 

 

 

 

Demineralized bone matrix contains collagen, BMP, TGF-beta, residual Ca, but NOT mesenchymal precursor cells. Live precursor cells are removed from allograft during processing. Calcium phosphate cement does not contain living cells. The only bone graft material that contain these live precursor cells are those containing autologous marrow- fresh autograft or bone marrow aspirate (answer 4).

 

 

 

 

 

 

 

What description below best describes galvanic corrosion?

 

 

 

 

 

 

  • Autologous iliac crest marrow aspirate
  • Calcium phosphate putty
  • Fresh-frozen allograft bone Recombinant bone morphogenic protein Demineralized bone matrix
     

    Corrent answer: 2

     

    A randomized controlled trial is the study design that can provide the most compelling evidence that the study treatment causes the expected effect on human health. The advantages of proper randomization include:

     

    It eliminates bias in treatment assignment, specifically selection bias and confounding.

     

    It facilitates blinding (masking) of the identity of treatments from investigators, participants, and assessors.

     

    It permits the use of probability theory to express the likelihood that any difference in outcome between treatment groups merely indicates chance.

     

    The reference by Poolman et al reviewed all JBJS articles from 2003-2004 and found that 31% of the studies used modified outcome questionnaires, and

    frequently did not describe how they modified the questionnaires. They also recommend that operating surgeons do not collect the outcome data, in order to minimize bias.

    Which of the following bone graft material contains live mesenchymal osteoblastic precursor cells?


    To increase treatment effect To prevent selection bias To generate enough power To achieve statistical significance To perform as-treated analysis
     

    Corrent answer: 5

     

    Ceramic bearings have not been documented to be an etiology for pelvic insufficiency fractures.

     

    The review article by Vanderschot describes that constrained liners used with total hip arthroplasty create a mechanical constraint that redistributes more force to the pelvic ring. Pelvic ring insufficiency fractures are increasing in incidence. They commonly present with groin pain in the elderly, but may also be found incidently on radiographs. CT scan, bone scan, and MRI can all be adjuncts to radiographs for diagnosis. Risk factors include osteoporosis, corticosteroid treatment, and rheumatoid arthritis.

    Illustration A shows a radiograph of a parasymphyseal fracture indicated by the arrows and is a common site for a pelvic insufficiency fracture.







     

    What is the primary purpose of utilizing a computer random number generator for patient randomization when designing a Level I therapeutic controlled trial?


    Osteoporosis Corticosteroid treatment Total hip replacement with constrained liner Rheumatoid arthritis Total hip replacement with ceramic bearings

    Corrent answer: 2

    Lead toxicity inhibits the effects of parathyroid hormone-related peptide (PTHrP). Previous animal studies have shown that lead results in decreased bone mineral density (BMD).

     

    Campbell et al performed a cohort study of children and found that subjects with high lead exposure had a significantly higher BMD than did subjects with low lead exposure. They hypothesize that the accelerated maturation of bone may ultimately result in a lower peak BMD being achieved in young adulthood, thus predisposing to osteoporosis in later life.





     

    Risk factors for insufficiency fractures of the pelvic ring include all of the following EXCEPT:

    factors?


    Thyroid stimulating hormone (TSH) Parathyroid hormone-related peptide (PTHrP) Human growth hormone (HGH) Insulin-like growth factor (IGF) Gonadotropin-releasing hormone (GnRH)
     

    Corrent answer: 5

     

    The question stem is describing osteoclasts, which resorb bone at the ruffled border and are inhibited by bisphosphonates. Osteoblasts are cuboidal cells that line along layers of immature osteoid to synthesize bone matrix and are not directly inhibited by bisphosphonates. Lymphocytes are small blue- appearing white blood cells with a large circular nucleus. Neutrophils are also known as polymorphonuclear white cells are white cells with multi-lobulated nuclei. Plasma cells are white cells with an eccentric nucleus with a clock face appearance.




     

    Lead toxicity affects growth by inhibiting which of the following growth


    neutrophils lympocytes plasma cells osteoblasts osteoclasts
     

    Corrent answer: 5

     

    In a locking plate fixation model, use of bicortical screws, parallel screw placement, increased plate length, increased number of screws, and screws placed at appropriate

    angles into the screw hole all serve to increase fixation construct stability.

     

    Far cortical locking screws lock into the plate as well as gain thread purchase on the far cortex, with a near cortex that does not touch the screw (whether via lack of threads or overdrilling of the near cortex). This acts to decrease

    construct stiffness.

     

    The referenced article by Haidukewych and Ricci reviews the concepts of locking plates, including biomechanics and technique.

     

    The referenced article by Kaab et al is a biomechanical study of locking plates that found significantly decreased construct stability if screw trajectory relative to the plate hole (i.e cross-thredading) varied > 5 degrees from the screw hole orientation.

     

    The referenced article by Sommer et al is a case series of locking plate failures and review of the concepts important to create a successful construct with these devices. They warn against inappropriate use of locking plates when standard plates can be used.

     

    Incorrect Answers:

    1-4: These all act to decrease construct stiffness.





     

    Microscopic description of "multinucleated giant cells that originate from monocyte lineage and possess a ruffled border" best describe which of the following cell types?


    Unicortical locking screws compared to bicortical locking screws Angular cross-threading screws into a plate Overdrilling the near cortex for the screw holes Far cortical locking screws Increased number of screws in the plate
     

    Corrent answer: 3

     

    Despite various reports documenting effects of low intensity ultrasound on living tissues, the exact mechanism of ultrasound on biological tissues remains largely unknown. It is most likely due to the mechanical energy of the sound waves that are transferred to tissues. Low frequency ultrasound in the region of 1.0 kHz may be capable of producing vibration and therefore strain across the cell wall (aka nanomotion at the fracture site).

     

    The study by Parvizi et al observed that rat chondrocytes were stimulated by an ultrasound signal. They found that intracellular Ca++ concentration increased and proteoglycansynthesis increased in response to stimulation. When intracellular calcium was chelated in their experiment, the increase in proteoglycan synthesis reduced significantly, indicating that the rising intracellular Ca++ concentration as a result of ultrasound stimulation functioned as an intracellular signal to increase proteoglycan synthesis.

     

    Leung et al showed that ultrasound had no significant effect on preventing bone loss in post-menopausal women.





     

    Assuming all other variables are the same, which of the following increases fixation construct stiffness in a locking plate model?


    decreasing intracellular calcium concentration decreasing temperature produces nanomotion at the fracture site decreases proteoglycan synthesis inhibits integrins

    1. 1 in 10,000,000

    2. 1 in 1,000,000

    3. 1 in 100,000

    4. 1 in 50,000

    5. 1 in 10,000

     

    Corrent answer: 2

     

    Hepatitis C & B, HIV, TB, and other bacterial infections have been recognized as important transmissible diseases in tissue and organ transplants. Retrospective studies, beginning in 1992, have demonstrated the ability of bone allografts from donors infected with these pathogens to seroconvert recipients and ultimately transmit the disease.

     

    The risk of viral transmission associated with blood properly screened for Hepatitis C is 1 in 100,000. Risk of transmission in Hepatitis B is 1 in 63,000, HIV is between 1 in 650,000 to 1 in 1,000,000 for blood transmission and allograft transmission rates are estimated by the blood transmission rates.

     

    In a current concepts review, Tomford reviews the current literature regarding disease transmission in musculoskeletal allografts. Currently two cases of HIV transmission have been documented in the United States. Current molecular biological efforts towards identification of potential infectious agents in allografts have shown promise in the early identification and prevention of disease transmission.

     

    Which of the following is a mechanism by which low-intensity pulsed ultrasound is reported to stimulate fracture healing?

     

    Corrent answer: 2

     

    The distance a screw traverses in a bone during fracture fixation is defined as the working length (or less commonly "working distance") of that screw.

     

    The referenced article is a good review of the biomechanics of plate and screw fixation, and covers important topics such as creating a high plate:span ratio and a low plate screw density.

     

    Incorrect Answers:

    1: Pitch: Distance between adjacent threads

    3: Screw Lead: Distance screw advances with each turn 4: Core Diameter: narrowest diameter of screw

    5: Area moment of Inertia: resistance to torsional force related to cube of radius





     

    What is the approximate risk of transmission of HIV in fresh- frozen allograft bone?


    Pitch Working length Screw lead Core diameter Area moment of inertia
     

    Corrent answer: 4

     

    Articulated dynamic-response prostheses have replaced SACH prostheses as the standard foot.

     

    The review article by Romo states the articulated dynamic prosthesis incorporates axial joints to provide articulation rather than solely relying on keel deformation to provide dorsiflexion in the stance phase of gait. Once the patient has reached the end of range of motion allowed by the articulation, the keel may deform with a spring-like dynamic response. The keel deforms under load, becoming a spring and allowing dorsiflexion to provide a springlike response for push-off. Most dynamic-response feet have a flexible keel. These characteristics make a flexible keel articulated dynamic-response prosthesis ideal for accommodating uneven surfaces. The addition of a sagittal split keel allows for more inversion and eversion allowing even better navigation of

    uneven terrain. A shortened keel is indicated for moderate level ambulators and lengthened keels are indicated for very high level demand athletes. A single axis foot only allows for dorsi/plantar flexion but has poor cosmesis and durability. Illustration A is an example of a single axis foot. Illustration B is an example of a rigid keel foot.









     

    The distance of bone traversed by a screw is defined as which of the following terms?

    solid-ankle, cushioned-heel (SACH) prosthetic foot. Which of the following modifications to the new prosthesis is most appropriate?


    Single axis foot Shortened keel articulated dynamic-response foot Lengthened keel articulated dynamic-response foot Sagittal split keel articulated dynamic-response foot Rigid keel nonarticulated dynamic-response foot
     

    Corrent answer: 2

    Synovial fluid is made by the fibroblast-like type B synovial cell. Type A synovial cells are important phagocytic cells. Recently a type C synovial cell has been identified and found to be an intermediate, non-functional cell, of unknown significance. Chondrocytes support the cartilage which immediately surrounds them. The fibrochondrocyte is the cell responsible for meniscal healing.

     

    In their study, Schwarz and Hills describe the production of synovial fluid as the key function of the type B synoviocytes. In addition, they identified the role of proteolipids in boundary lubrication within synovial fluid and provide a comparison between the function of such proteolipids in the synovial fluid with their function in the lung. They hypothesize based on data presented in this study that exogenous administration of surface-active phospholipids may be beneficial in the treatment of arthritis.





     

    A 44-year-old male with transtibial amputation is interested in doing more hiking as a hobby, however he has had difficulty negotiating uneven terrain with a


    Type A synovial cell Type B synovial cell Type C synovial cell Chondrocyte Fibrochondrocyte
     

    Corrent answer: 3

     

    LCDC plates have less contact with the bone as compared to the standard dynamic compression plates. The decreased surface area leads to less contact- induced osteopenia.

     

    The Perren article describes the advantages of locked plating when viewed as an internal fixator. In other words, the locked plate functions like an external fixator yet it is close to the bone and in situ, i.e. an internal fixator. The negatives of dynamic compression plating include the osteopenia that can develop because dynamic plating relies on bone plate friction whereas the locking plate relies on the screws and screw placement. Perren states that

    “The biological aspects of damage to the blood supply, necrosis and temporary porosity explain the importance of avoiding extensive contact of the implant with bone.”





     

    A healthy human knee normally contains approximately 2 milliliters of synovial fluid. What cell produces synovial fluid?


    Less implant-bone and fracture gap micromotion More fracture site compression Less implant-bone contact induced osteopenia Stiffer fracture fixation construct More stress shielding

    Corrent answer: 1

     

    Using silicone arterial dye injection in a cadaveric femur model, Farouk et al. studied the vascular disruption of conventional plate osteosynthesis (CPO) with minimally invasive plate osteosynthesis (MIPO). All MIPO specimens showed intact perforating and nutrient arteries, whereas the CPO specimens had a variable incidence of vessel disruption. The MIPO group demonstrated better periosteal perfusion in each of the cadavers and improved medullary perfusion

    in 70 percent of the MIPO specimens compared with the CPO specimens. Based on this evidence, they concluded that MIPO may be more advantageous biologically than the traditional method.





     

    Limited contact dynamic compression (LCDC) plates have what advantage over standard dynamic compression plates?


    standard plating results in greater compromise to both medullary and periosteal bloodflow standard plating results in greater compromise to periosteal bloodflow only standard plating results in greater compromise to medullary bloodflow only standard plating results in less compromise to both medullary and periosteal bloodflow there is no difference between the two techniques with respect to periosteal and medullary bloodflow
     

    Corrent answer: 4

     

    The effect of bisphosphonates is focused directly on the osteoclast. Bisphosphonates preferentially bind to mineral exposed by bone-resorbing

    osteoclasts. The osteoclasts resorb the bone and take in the bisphosphonate. The net result of bisphosphonates is to decrease bone resorption and decrease bone turnover (why #5 is incorrect).

    There are two groups of bisphophonates: non-nitrogen containing and nitrogen-containing. Non-nitrogen bisphosphonates (e.g. clodronate and etidronate) induce osteoclast apoptosis. Clodronate creates a toxic analog of adenosine triphosphate, which targets the mitochondria. For nitrogen- containing bisphosphonates, farnesyl diphosphate synthase (important in the cholesterol biosynthetic pathway) is the direct intracellular target. Its inhibition suppresses protein geranylgeranylation, an essential step in the cellular processes required for osteoclastic bone resorption. Answer #3 would be the correct answer if the question asked about nitrogen-containing bisphosphonates.

     

    The reference by Reszka et al reviews the above information about the mechanism of action of bisphosphonates as well as reviews their indications.

     

    The reference by Rodan et al is a 2003 review of bisphosphonate use in osteoporosis and reviews the use of this class of medications at that time.





     

    Which statement is true regarding standard open plating techniques compared to minimally invasive submuscular plating techniques?


    Post-translational carboxylation of vitamin K dependent proteins Decreased osteoblastic acivity Inhibits farnesyl diphosphate synthase in osteoclasts Increased osteoclast apoptosis Increases bone turnover
     

    Corrent answer: 3

     

    Increased knee extension during stance can be a result of a heel that is too soft or a foot that placed is too anterior. Posterior placement of the prosthetic foot or a heel that is too hard will increase knee flexion/instability at heel strike. A prosthetic foot that is too far inset produces varus strain, pain (proximal/medial and distal/lateral) at the socket, and a circumducted gait. A

    prosthetic foot that is too far outset will produce valgus strain, socket pain that is proximal/lateral and distal/medial, and a broad-based gait.








     

    What is the mechanism of action of non-nitrogen containing bisphosphonate (e.g. clodronate and etidronate) medications?


    Varus strain Socket pain located medial and proximal Increased knee extension during stance Socket pain located lateral and distal Circumducted gait

    Corrent answer: 4

     

    Definition of genetic imprinting: the allele that is expressed is determined solely on which parent contributes it. In Angelman syndrome the defective gene comes from the mother, leading to epilepsy, tremors, and smiling facies. In Prader-Willi syndrome the defective gene comes from the father resulting in hypotonia, obesity, hypogonadism. These are classic examples of genetic imprinting in humans. Usually, diseases resulting from defective structural proteins are autosomal dominant, while defective enzymes are autosomal recessive.





     

    A 27-year-old patient comes in for a new prescription for his below knee amputation prosthesis because it is not fitting properly. All of the following are complaints and examination findings consistent with a prosthetic foot that is placed too far inset EXCEPT:


    Sex-linked dominant transmission Y-linked dominant transmission X-linked dominant transmission Genomic imprinting Mendelian inheritance

    Corrent answer: 1

    Turchin et al assessed the outcome of two groups of matched polytrauma patients, with the only difference being the presence of a foot injury in Group

    1. They used three outcome tools, SF-36, WOMAC, and Modified Boston Children's Hospital Grading System to evaluate the the two groups at a minimum of 2 years from injury. The foot injury group, including all types of foot fractures, had a poor outcome when using any of these measures. Turchin concludes that “Foot injuries cause significant disability to multiply injured patients. More attention should be given to these injuries, and more

    aggressive management should be considered to improve the outcome of this group of multiply injured patients.”

     

    Tran et al compared polytrauma patients with foot injuries to 14 polytrauma patients without foot injury at a minimum of 12 months from injury. The AAOS lower limb and foot and ankle outcomes data collection questionnaire showed significantly lower scores in the foot injury group. The Constant score is used for evaluation of shoulder conditions.

     

    Prader-Willi syndrome and Angelman syndrome are examples of a genetic phenomenon by which certain genes are expressed in a parent-of-origin-specific manner. What is the name of this genetic phenomenon?

    matched polytrauma patient without a foot injury?



    Lower mean Short Form 36 (SF-36) score Higher mean score on the AAOS lower limb and foot and ankle outcomes questionnaire Equivalent score on the Western Ontario and McMaster Universities Osteoarthritis Index (WOMAC) Lower Constant score Higher score on the Western Ontario and McMaster Universities Osteoarthritis Index (WOMAC)
     

    Corrent answer: 4

     

    This patient with hemophilia A is presenting with an intramuscular hematoma surrounding the iliacus muscle which is likely to cause femoral nerve compression. Paresthesias would be expected in the L4 nerve distribution.

     

    Excessive bleeding into joints and muscles is a common manifestation of hemophilia. The iliacus muscle is a frequent site of hemorrhage in patients with severe or moderate hemophilia. Intramuscular hematoma of the iliacus muscle is likely to occur following play or sporting events that include forceful contraction of the hip flexor muscles. As the hematoma expands, it may

    compress the adjacent femoral nerve, potentially resulting in complete femoral nerve palsy. Femoral nerve compression typically includes paresthesias in the distribution of the terminal saphenous nerve branch.

    Gilbert et al. review the complex relationship between recurrent bleeding, synovitis, and the development of arthritis in the patient with hemophilia. They discuss both conservative and surgical treatment modalities in these patients and recommend arthroscopic synovectomy for the knee and ankle joints. They conclude that the greatest risk to these procedures is a decreased range of motion.

     

    Kuo et al. reports on a fourteen-year-old healthy boy with an 11-day history of pain and weakness in the right lower limb following a fall. They report pain in the right lower extremity, numbness of the anterior aspect of the right thigh and medial border of the right leg and foot, inability to ambulate and

    weakened quadriceps muscle strength. MRI revealed an iliacus hematoma with a complete femoral nerve palsy. He underwent CT-guided percutaneous drainage for decompression with complete resolution of the palsy.

     

    Illustration A is a diagram of dermatomal distribution. Illustration B shows the lumbar plexus demonstrating the intimate relationship of the femoral nerve to the iliacus muscle.

     

    Incorrect Answers:

    Answers 1-3, 5: An intramuscular hematoma of the iliacus would most likely cause compression of the surrounding femoral nerve. These other lumbar nerves would be less likely to result in paresthesia.








     

    A 45-year-old male trauma patient presents with multiple extremity injuries including the foot injury shown in Figure A. The foot fracture is treated surgically, and heals without any initial complications. At a minimum of 12 months, this patient will be expected to have which of the following scores compared to a


    Genitofemoral Iliohypogastric Ilioinguinal Femoral Obturator

    Patients with pauciarticular juvenile rheumatoid arthritis (JRA), specifically the subgroup with elevated antinuclear antibody (ANA) titers, are associated with the highest incidence (~75%) of anterior uveitis. As a result, referral for an ophthalmology consultation is recommended.

     

    Pauciarticular JRA is the most common subgroup of JRA and typically presents between the ages of 2 to 4 years with mild swelling of one to four joints. The diagnosis is typically one of exclusion as laboratory studies, including erythrocyte sedimentation rate and rheumatoid factor, are usually within normal limits. In JRA, iridocyclitis, a type of anterior uveitis typically occurs following the onset of synovitis but may precede the joint symptoms. This iridocyclitis is frequently indolent but requires immediate ophthalmologic consultation for a slit-lamp examination because if left untreated, anterior uveitis may progress to loss of vision.

     

    Foeldavri et al. review JRA anterior uveitis. They report an overall incidence of

    10%, but this is dependent on the JRA subtype. They noted that a large proportion of children with JRA develop uveitis in the first year of disease and

    73-90% after 4 years. They state that early age of JRA onset, oligoarticular subtype, and ANA reactivity are the main risk factors for the development of uveitis. They conclude that JRA-associated uveitis is important to recognize and treat early to prevent any visual damage.

     

    Hawkins et al. review bilateral chronic anterior uveitis in JRA. They report that female gender, oligoarthritis, and presence of antinuclear antibodies are risk factors.

    They report on treatment options, including the use of biologics. They conclude that stepwise immunomodulatory therapy is indicated, with new biologic drugs being used in cases of refractory uveitis.

     

    Incorrect Answers:

    Answers 1: Down syndrome does not require ophthalmologic consultation for slit lamp examination to evaluate for iridocyclitis, a type of anterior uveitis. Answer 2: Marfan syndrome is associated with anterior lens dislocation, not anterior uveitis

    Answer 3: Hunter syndrome, and other mucopolysaccharidoses, are associated with corneal clouding, not anterior uveitis

    Anterior 4: Pompe disease is a glycogen storage disease which may lead to ptosis (drooping of the upper eyelid), not anterior uveitis







     

    A 9-year-old male with hemophilia A presents with severe groin pain, parasthesias over the medial aspect of the distal tibia, and difficulty ambulating several hours after a soccer game. He is believed to have an intramuscular hematoma surrounding the iliacus muscle. Which nerve is MOST likely to be compressed?


    Down syndrome Marfan Syndome Hunter Syndrome Pompe Disease Pauciarticular juvenile rheumatoid arthritis with positive ANA Corrent answer: 5



     

    Which of the following conditions places the patient at highest risk for anterior uveitis and necessitates referral to an ophthalmologist?

     

    Corrent answer: 5

     

    Salmonella is a classic cause of osteomyelitis in patients with sickle cell disease.

    Sickle cell disease is a genetic disorder of hemoglobin synthesis. The disease occurs in two phenotypes: sickle cell anemia (most severe) and sickle cell trait (most common). The two most common causes of osteomyelitis in children with sickle cell disease are

    Staphylococcus aureus and Salmonella. Although S. aureus is the most common cause of osteomyelitis in the general population, the literature varies on which is the most common in patients with sickle cell disease. The increased risk in these patients may be associated with gastrointestinal microinfarcts, poor circulation of blood in bone, and splenic infarcts that predispose patients to infection by encapsulated bacteria (i.e., Salmonella).

     

    Piehl et al. analyzed records of seven hundred seventeen patients with sickle cell disease treated over a thirteen-year period. They identified and retrospectively reviewed sixteen cases of osteomyelitis in fifteen patients. The authors found Salmonella to be the causative organism in thirteen cases with Proteus mirabilis, Escherichia coli, and Staphylococcus aureus all affecting one patient each. The authors report the annual incidence of osteomyelitis in their series as 0.36%.

     

    Givner et al. reviewed sixty-eight cases of osteomyelitis in children with sickle cell disease and positive cultures over a ten year period. Of the sixty-eight, 50 (75%) yielded Salmonella and Staphylococci was isolated 7 (10%). In

    addition, the authors report non-speciated gram-positive cocci were isolated in

    11 (16%), non-speciated gram-negative rods in 5 (7%), and non-specified bacteria in 2 (3%). The authors conclude Salmonella is the most common pathogen causing osteomyelitis in patients with major sickle hemoglobinopathies.

     

    Epps et al. reviewed fifteen patients with sickle cell disease and osteomyelitis. Staphylococcus aureus was isolated in eight cases (53%), Salmonella in six (40%), and Proteus mirabilis in one (7%). The authors conclude S. aureus, not Salmonella, may be the most common cause of osteomyelitis associated in patients with sickle-cell disease.

     

    Figure A demonstrates an osteolytic lesion of the distal tibia and Figure F demonstrates sickle-shaped erythrocytes.

     

    Incorrect Answers

    Answers 1-4: These conditions are not classically associated with an increased risk of salmonella osteomyelitis.

    Figure B demonstrates round plasma cells with eccentric nuclei and clock face organization of chromatin, characteristic of multiple myeloma. Figure C demonstrates uniform spindled cells with stromal calcifications, consistent with monophasic synovial sarcoma. Figure D demonstrates numerous giant cells consistent with a giant cell tumor. Figure E demonstrates pseudo-rosettes, which may be present in Ewing’s sarcoma.





    Figure B Figure C Figure D Figure E Figure F
     

    Corrent answer: 3

     

    Low toughness is a disadvantage of ceramic bearings in total hip arthroplasty.

     

    Ceramic is a non-metal that demonstrates excellent wear characteristics when used with polyethylene in total hip arthroplasty. Although it has a high Young's modulus, it has a low fracture toughness. Subsequently, ceramic is poorly resistant to crack formation. In contrast, UHMWPE has a high fracture toughness because of the presence of very long hydrocarbon chains.

    Santavirta et al. review alternative bearing materials to improve wear in total hip arthroplasty. Alumina ceramics are noted to be biostable and bioinert. The best wear properties are noted with ceramic-on-ceramic bearings. For current ceramic constructs, fracture risk is less than 1 per 1000.

     

    Lang et al. review the use of ceramics in total hip replacement. The authors note that ceramic has high compressive strength and high wettability. Low fracture toughness and linear elastic behavior increase the risk of breakage of ceramic components under stress. Processing improvements, enhanced head- neck interfaces and liner modifications have lead to a decrease in the rate of ceramic fracture.

     

    Illustration A shows a compromised ceramic head as a manifestation of the low fracture toughness of the material.

    Incorrect Answers:

    Answers 1, 2, 4, 5: Low fracture toughness is a characteristic of ceramic that risks component compromise during placement.









     

    An 8-year-old African American female presents with lower extremity pain and subjective fever. On exam there is tenderness about the distal tibia. Further workup reveals elevated inflammatory markers and a lytic lesion (Figure A). An aspirate is obtained and cultures grow Salmonella. Additional investigation is most likely to reveal which of the following findings (Figure B-F)?


    Cobalt chromium Titanium Ceramic Polyethylene Stainless steel
     

    Corrent answer: 4

     

    An ideal fluid film lubrication regime minimizes friction. A larger head size results in a greater development of full-film lubrication due to the increased relative sliding velocity of the larger bearing surfaces. Increased surface roughness inhibits the formation of the film lubrication. The most important factor influencing the predicted lubrication film thickness

    has been found to be the radial clearance between the ball and the socket.

     

    Jin et al report that slight clearance, not complete congruence, is optimal for formation of the optimal fluid film lubrication. They note that full fluid film lubrication may be achieved in these hard/hard bearings provided that the surface finish of the bearing surface and the radial clearance are chosen correctly and maintained.

     

    Dumbleton reviewed the literature of metal-on-metal THA and concluded that the current literature does not show any clinical benefit of metal-on-metal compared to metal on poly. Metal-on-metal has been shown to have higher metal ion level in blood, and measurement of these levels is recommended to help identify those at risk of adverse effects from metal on metal prostheses.





     

    Low toughness is a disadvantage of which of the following bearing surfaces used in total hip arthroplasty?


    Larger femoral head Slight clearance of fit between the acetabular cup and the femoral head Minimal surface roughness of the femoral head Complete congruence of fit between the acetabular cup and the femoral head Minimal surface roughness of the acetabular cup
     

    Corrent answer: 5

     

    This attending did not fully disclose that the resident would be performing the cementing portions of the case unsupervised. This represents an ethically unsound scenario as the patient was misled regarding involvement of the resident in their surgery.

    The informed consent process is grounded in the ethical principle of autonomy. Informed consent represents a shared decision making process where a

    patient understands all the risks and benefits of a surgery fully and makes an informed decision. However, the patient's choice of surgeon is felt to be critical

    to the informed consent process and any variation from that surgeon performing the surgery should be discussed explicitly. A surgeon who performs surgery or part of surgery on the patient without prior consent may be held liable for battery.

     

    Kocher presents three cases demonstrating the spectrum of "ghost surgery". They state the substitution of an authorized surgeon with an unauthorized surgeon or allowing surgical trainees to operate without appropriate guidance constitutes "ghost surgery".

    Deviation from what is explicitly discussed has been justified in an emergency scenario or if the treatment is aimed at an overall condition.

     

    Bhattacharyya et al reviewed malpractice claims for factors that positively correlated with successful defense. They found that those who performed informed consent in the office had lower risk of malpractice payment. They conclude surgeons can decrease their risk of malpractice claims by performing informed consent in the office and documenting the discussion.

     

    Incorrect Answers:

    Answer 1: Attending surgeon presence for the surgical timeout is important  but presence during critical portions is required for billing purposes under CMS guidelines.

    Answer 2: Having an immediately available surgeon to assist should the primary surgeon be unable to is required for billing purposes under CMS guidelines.

    Answer 3: Discussion of resident supervision should be the same regardless whether the surgery is performed at an academic medical center or at another hospital site.

    Answer 4: Transparent disclosure of the possibility of overlapping surgery can facilitate ethically sound overlapping surgery. Concurrent surgery is when critical portions of surgeries occur simultaneously and are





     

    Which of the following features of metal-on-metal total hip arthroplasty does not allow for improved fluid film lubrication between the components?


    This practice is ethically sound as the attending was present for the surgical timeout This practice is ethically sound as long as another attending surgeon was immediately available to assist This practice is ethically sound as the surgery was performed at an academic medical center This practice is ethically unsound as this represents overlapping surgery This practice is ethically unsound as the patient was misled
     

    Corrent answer: 3

     

    The patient sustained a fragility fracture with lab work consistent with primary hyperparathyroidism.

    Hyperparathyroidism is commonly caused by increased activity of the parathyroid glands resulting in high levels of PTH. Increased circulating levels of PTH leads to calcium being "sucked" out of bone and into the serum. This

    alteration in calcium hemostasis leads to low-density bone and a predisposition to fragility type fractures. When present, lab values are much different from standard age-related osteoporosis. Furthermore, referral to medical and surgical endocrinology specialists for directed treatments may improve overall bone quality and prevent further fragility fractures.

     

    Fraser summarizes primary and secondary hyperparathyroidism. He describes the normal physiologic response to low calcium of an increase in PTH. Increased PTH has three downstream effects of increased tubular resorption of calcium by the kidneys, increased osteoclast activity to harvest calcium from bone, and increased active vitamin D levels leading to increased bowel absorption of calcium.

     

    Singhal et al. reviewed hyperparathyroidism and what the orthopedic surgeon should know. They state when a patient presents with a pathologic fracture and elevated serum calcium levels, an appropriate lab workup for hyperparathyroidism should be done. They stated when surgery is needed for hyperparathyroidism and fracture, surgery can safely be performed simultaneously as demonstrated by 3 case examples.

     

    Figure A exhibits a left femoral neck fracture, which is a fragility fracture associated with poor bone density. Illustration A is a figure from Fraser's article exhibiting the

    feedback loop from the hypothalamus, pituitary, adrenal/glandular axis.

     

    Incorrect answers:

    Answer 1: An age-related decrease in bone mass describes osteoporosis which would be expected to have normal labs.

    Answer 2: Decreased PTH production following thyroidectomy describes hypoparathyroidism.

    Answer 4: Hypophosphatasia presents in childhood and labs are typically normal with the exception of low serum alkaline phosphatase.

    Answer 5: Vitamin C deficiency causes growth plate abnormalities at zone of provisional calcification of the physis.





     

    Prior to undergoing a total knee arthroplasty at an academic medical center a patient is told during informed consent by the attending surgeon that resident involvement in the case will be limited to retracting. During the case the attending is present up to trialing of the selected components. The surgeon leaves prior to cementing to start trialing components in another case while the chief resident remains alone in the room for the completion of the case. Which of the following is true regarding the ethics of this practice?



    An age-related decrease in bone mass Decreased PTH production following thyroidectomy Increased PTH secondary to adenoma Genetic defect leading to decreased inorganic phosphate production Vitamin C deficiency
     

    Corrent answer: 4

     

    Patients in factorial randomized control trials (RCT) are assigned to groups that receive a specific combination of interventions and non-interventions.

    In factorial RCTs, patients are randomized to groups receiving treatment A and B, treatment A or control, treatment B or control, or no treatment. This study design is useful because two interventions can be assessed with the same

    study population and any interaction between the treatments can be determined (for example, does treatment A work differentially when combined with treatment B). Other randomized control trial designs include parallel, cluster, and crossover. Parallel studies are performed by having two or more groups that exclusively have one intervention without group overlap.

    Crossover studies have each group receive each intervention in a random sequence. Cluster design studies have pre-existing groups of participants

    (such as schools, or cities) that are randomly selected to receive or not receive an intervention.

     

    Karlsson and the International Society of Arthroscopy, Knee Surgery and Orthopaedic Sports Medicine published an exhaustive guide to research for evidence-based medicine in a step-wise fashion. They cover levels of evidence, design for randomized control trials and the CONSORT checklist. They also describe proper study design of cohort, case- control, case series, systematic review, meta-analysis studies. The second half of the guide discusses appropriate outcome measures, statistical analyses, and data interpretation, reporting complications, and concludes with steps to writing a scientific article.

     

    Incorrect Answers:

    Answer 1: Parallel RCTs are when each study group receives or does not receive an intervention.

    Answer 2: Cluster RCTs are when pre-existing groups are assigned to receive or not receive an intervention.

    Answer 3: This is not an RCT. Cohort studies are longitudinal studies looking at a known exposure for a possible later outcome (such as a disease).

    Answer 5: Crossover RCTs are when each study group receives or does not receive an intervention in a random sequence.





     

    A 66-year-old woman falls from standing and sustains the injury shown in Figure A. Her most recent T score was -1.9, 3 months prior to presentation. Preoperative lab work reveals elevated serum calcium, elevated alkaline phosphatase, decreased serum phosphorus, and elevated parathyroid hormone (PTH). Which of the following correctly describes the underlying etiology of her osteopenia?


    Parallel Cluster Cohort Factorial Crossover
     

    Corrent answer: 3

     

    The most recent update of the CDC guidelines for the prevention of SSI issues a category IA strong recommendation stating that "in clean and clean- contaminated procedures, do not administer additional antimicrobial prophylaxis doses after the surgical incision is closed in the operating room, even in the presence of a drain."

    The previous 2002 CDC guidelines for the prevention of SSI focused on three performance parameters: (1) initiation of parenteral antibiotics within 1 hour of the surgical incision, (2) selection of an appropriate antibiotic, and (3) discontinuation of antibiotics within 24 hours. The most recent updated 2017

    CDC guidelines for the prevention of SSI has several notable changes with an emphasis that additional doses of antibiotics after initial prophylaxis are no longer recommended.

     

    Berrios-Torres et al. review the 2017 updates to the CDC guidelines for prophylaxis against SSI. Strong recommendations include that in clean and clean-contaminated cases, additional antimicrobial prophylaxis should not be administered after the surgical incision is closed in the operating room, even in the presence of a drain. Furthermore, the authors discuss that there is no evidence that re-dosing intraoperative antibiotics or continuation of antibiotics until surgical drains have been removed provides any additional protection against surgical site infection.

     

    O'Hara et al. highlights the key updates in the most recent CDC guidelines for prevention of SSI. The authors present specific suggestions for translating these recommendations into evidence-based policies and practices. They conclude that the implementation of new and existing guidelines in SSI prevention requires thoughtful and careful collaboration with several inter- professional and interdisciplinary teams.

     

    Incorrect Answers:

    Answer 1: The 2017 CDC guidelines did not identify sufficient evidence to evaluate the benefits and harms of intraoperative re-dosing of parenteral prophylactic antibiotics.

    Answer 2, 4 and 5: A strong (Category 1A) recommendation noted in the 2017 CDC guidelines states that in clean and clean-contaminated cases, additional prophylactic antimicrobial doses after the surgical incision is closed in the operating room should not be administered, even in the presence of a drain.





     

    Which of the following study designs describes a randomized controlled trial in which two interventions are applied separately or in combination to study groups?


    Intra-operative re-dosing of antibiotics is recommended for longer surgical procedures Antibiotics should be continued for 24 hours post-operatively in the presence of a surgical drain In clean-contaminated procedures, a single dose of peri-operative antibiotics is sufficient for SSI prophylaxis In clean procedures, duration of post-operative antibiotics depends on duration of surgery Antibiotics should be continued until all surgical drains are removed

    Corrent answer: 5

     

    The patient has an allergy to cephalosporins and a history of an MRSA infection. Of the choices listed, vancomycin is the best preoperative antibiotic for this patient.

     

    The choice of preoperative antibiotics is of great interest given the large

    medical and economic cost of periprosthetic infections. Standard preoperative prophylaxis in patients undergoing total joint arthroplasty consists of cefazolin or cefuroxime. In patients with beta-lactam allergies, the treatment options include clindamycin or vancomycin. Vancomycin is often the antibiotic of choice given it's higher efficacy with regard to MRSA prevention. In those patients who are considered at risk for MRSA infection and a beta-lactam allergy, vancomycin can be supplemented with an aminoglycoside (gentamicin) or aztreonam.

     

    Bratzler et al. review antimicrobial prophylaxis for surgery and state for orthopedic joint replacement procedures cefazolin or cefuroxime is the recommended antibiotic. For patients with a confirmed beta-lactam allergy, they recommend vancomycin or clindamycin. They also state antibiotics should be stopped within 24hrs after surgery.

     

    Dellinger et al. review antibiotics for surgical prophylaxis. They state the standard antibiotics for orthopedic procedures are cefazolin or cefuroxime. They state if there is also a concern for MRSA infection vancomycin can be added in addition to the above antibiotics.

     

    Incorrect Answers:

    Answer 1: This describes the aminoglycoside (gentamycin, neomycin, etc) family of antibiotics which would not be the preferred prophylactic antibiotic in this patient.

    Answer 2: This describes the macrolide (erythromycin, azithromycin, etc) family as well as clindamycin which would not be the preferred prophylactic antibiotic in this patient.

    Answer 3: This describes the fluoroquinolone family of antibiotics which would not be the recommended antibiotic in this patient.

    Answer 4: This describes the cephalosporin family of antibiotics which this patient has a documented severe allergy to.





     

    Which of the following is STRONGLY recommended by the most recent (2017) Centers for Disease Control and Prevention (CDC) Guidelines with regard to antimicrobial prophylaxis for the prevention of surgical site infection (SSI)?


    Bactericidal through irreversible binding to 30S ribosomal subunit Bacteriostatic through reversible binding to 50S ribosomal subunit Bactericidal through inhibitition of DNA gyrase enzyme Bactericidal by disrupting peptioglycan cross-linkage and does not cause red man syndrome Bactericidal by disrupting peptioglycan cross-linkage and may cause red man syndrome
     

    Corrent answer: 3

     

    Clindamycin is a bacterial protein synthesis inhibitor by inhibiting ribosomal translocation at the 50S subunit.

     

    Clindamycin is primarily bacteriostatic but may be bactericidal at higher concentrations.

    Side effects of clindamycin may include a hypersensitivity reaction and pseudomembranous colitis. Resistance to clindamycin is conferred by a plasmid that alters the 50s ribosome binding site for clindamycin. The D- zone test is used to determine whether an organism has inducible resistance

    to clindamycin.

     

    Marcotte and Trzeciak published a review on community-acquired methicillin- resistant Staphylococcus aureus (CA-MRSA). They reported that CA-MRSA

    does not have predictable susceptibility to clindamycin. They conclude that clindamycin also presents a risk for the development of Clostridium difficile colitis and inducible clindamycin resistance for which a D-zone test should be performed when culture results reveal erythromycin resistance.

     

    Steward et al. performed a lab study to determine the efficacy of testing for induced clindamycin resistance in erythromycin-resistant Staphylococcus aureus. They reported that resistance to erythromycin and clindamycin can occur through methylation of their ribosomal target site (50s), which is mediated by erm genes. They conclude that disk diffusion is the preferred method for testing S. aureus isolates for inducible clindamycin resistance.

     

    Incorrect Answers:

    Answer 1: Cefazolin inhibits cell wall synthesis via competitive inhibition of the transpeptidase enzyme.

    Answer 2: Ciprofloxacin is a fluoroquinolone. It inhibits DNA synthase by inhibiting DNA gyrase.

    Answer 4: Doxycycline is a tetracycline and inhibits the 30S ribosomal subunit. Answer 5: Vancomycin is an inhibitor of cell wall synthesis via disruption of peptidoglycan cross- linkage.





     

    A 68-year-old man is scheduled to undergo total hip arthroplasty. He states he had an anaphylactic reaction after taking cefazolin for an MRSA hand infection 10 years ago. Which of the following best describes the preoperative antibiotic that should be administered for this patient?



    Cefazolin Ciprofloxacin Clindamycin Doxycycline Vancomycin

    Corrent answer: 4

     

    Advanced glycation end-products (AGEs) cause excessive cross-linking of collagen in aging articular cartilage. As a result, the stiffness is increased.

     

    AGEs are produced by spontaneous nonenzymatic glycation of proteins when sugars (glucose, fructose, ribose) react with lysine or arginine residues. The most abundant matrix protein in cartilage is Type II collagen. AGEs cause changes to the aging cartilage matrix and the aging chondrocyte. The increased cross-linking of Type II collagen results in an increase in cartilage stiffness (i.e. increase in the modulus of elasticity) and an increase in brittleness (i.e. less strain needed to go from the yield point to the fracture point on the stress-strain curve). As a result of the change in the aging cartilage’s biomechanical properties, it's susceptible to fatigue failure. Additionally, AGEs decrease the anabolic response of chondrocytes from autocrine signaling via TGF-beta, IGF-1, BMP-7, and OP-1. These two initial mechanisms contribute to aging cartilage to eventually lead to the development of osteoarthritis.

     

    Li et al. reviewed age-related changes in cartilage and seek to define the different

    mechanisms between aging cartilage and osteoarthritis. They state that with AGEs, there is excessive collagen cross-linking increases cartilage stiffness, while shortening/degradation of aggrecan leads to loss of sugar side chains and water-binding ability. Additionally, increased levels of AGEs are associated with a decline in anabolic activity. They state that these changes to cartilage make it more vulnerable to damage and therefore the onset of osteoarthritis. This is contrast to the initial steps in the mechanism of osteoarthritis which is characterized by cell proliferation, formation of chondrocyte clusters, increased synthesis of irregular cartilage matrix, and eventually a pro-catabolic and pro-inflammatory state that results in an imbalance in cartilage homeostasis and cartilage matrix breakdown.

     

    Anderson et al. reviewed the relationship between osteoarthritis and aging.

    They state that knee cartilage thins with aging, especially on the femoral and patellar sides, suggesting a gradual loss of cartilage matrix. AGEs formation leads to modification of type II collagen by cross-linking of collagen molecules, increasing stiffness and brittleness and increasing susceptibility to fatigue failure. Furthermore, describe the senescent phenotype of the chondrocyte

    and its similarities with osteoarthritic chondrocyte phenotype.

     

    Incorrect Answers:

    Answer 1: Unresurfaced patellar cartilage will show similar changes to the other compartments prior to resurfacing. Thinning is a result of loss of cartilage matrix, rather than accumulation of AGEs.

    Answer 2: AGEs are characteristic of aging cartilage (rather than RA). It is found in articular cartilage (rather than synovium).

    Answer 3: AGES are not characteristic of hemarthrosis and do not result in cartilage staining.

    Answer 5: AGEs are found in aging cartilage not in osteoarthritis. An increase in AGEs in aging cartilage increases the stiffness of cartilage.







     

    Which of the following antibiotics works by binding to the 50S ribosomal subunit?

    Increased AGEs are characteristic of unresurfaced patellar cartilage after total knee arthroplasty and results in articular cartilage thinning. Increased AGEs are characteristic of rheumatoid arthritis and results in synovial thickening. Increased AGEs are characteristic hemarthrosis and results in articular cartilage staining. Increased AGEs are characteristic of aging articular cartilage and results in increased articular cartilage stiffness. Increased AGEs are characteristic of osteoarthritis and results in decreased articular cartilage stiffness.
     

    Corrent answer: 4

     

    The patient has clinical signs and symptoms of gout. Figure D would correspond to this diagnosis as it shows negatively birefringent needle-shaped monosodium urate crystals.

     

    Gout is an idiopathic disorder of nucleic acid metabolism that leads to hyperuricemia and deposition of monosodium urate crystals, most commonly in the joints of the lower limb (knee, ankle, and classically the 1st metatarsophalangeal joint). Diagnosis can be confirmed with joint arthrocentesis revealing negatively birefringent needle-shaped crystals. Treatment of acute gout flares is generally comprised of NSAIDs and colchicine, and chronic gout is treated with allopurinol to prevent the build-up

    of uric acid.

     

    Shmerling et al. prospectively analyzed the synovial fluid test results of 100 consecutive patients undergoing diagnostic arthrocentesis. They noted that synovial fluid white blood cell count (WBC) and the percentage of polymorphonuclear cells performed well as discriminators between inflammatory and noninflammatory diseases. Given the diagnostic value of synovial WBCs, the authors concluded that ordering of chemistry studies of synovial fluid should be discouraged because they are likely to provide misleading or redundant information.

     

    Chiodo et al. review the use of intra-articular aspiration and injections for both diagnosis and treatment of disorders of the lower extremity such as infectious arthritis, gout, pigmented villonodular synovitis (PVNS), rheumatoid arthritis, and hemophilia. The authors discuss the importance of knowledge of regional anatomy, procedural indications, and appropriate techniques for successful aspiration/injection. The authors review safe and effective aspiration and injection techniques for the lower extremity, including the hip, knee, foot, and ankle.

    Figure A reveals hemosiderin stained multinucleated giant cells consistent with PVNS. Figure B is a gram stain revealing gram-positive cocci in clusters consistent with Staphylococcus aureus. Figure C reveals rhomboid-shaped, positively birefrigerant crystal consistent with calcium pyrophosphate/pseudogout. Figure D reveals negatively birefringent needle- shaped crystals of monosodium urate/gout. Figure E reveals a collection of histiocytes and inflammatory cells around prominent intimal hyperplasia.

     

    Incorrect Answers

    Answer 1: Figure A reveals hemosiderin stained multinucleated giant cells consistent with PVNS

    Answer 2: Figure B is a gram stain revealing gram-positive cocci consistent with septic arthritis

    Answer 3: Figure C reveals rhomboid-shaped positively refrigerant calcium pyrophosphate crystals consistent with pseudogout

    Answer 5: Figure E reveals a collection of histiocytes and focal inflammatory cells consistent with rheumatoid arthritis





     

    An increase in advanced glycation end-products (AGEs) is characteristic of which of the following clinical conditions and results in which pathologic process?




    Figure A Figure B Figure C Figure D Figure E
     

    Corrent answer: 2

     

    Regardless of the number of level I studies included in a systematic review, having one study with <80% follow-up decreases the level of evidence for this review from level I to level II.

    After classifying the type of study (e.g. therapeutic study, prognostic study, diagnostic study, economic analysis, or decision analysis) the “level of evidence” is then determined. The level of evidence (on a scale of I through V) for medical research is determined. It is important to consider the characteristics of a study’s design. This would include the percent follow-up, utilization of a control group, presence of blinding, heterogeneity of results, and process of randomization. Specific to meta-analyses and systematic reviews, it is important to know that the lowest quality study used in the review determines the level of evidence. In evidence-based medicine, higher levels of evidence have a larger impact on clinical recommendations.

     

    Bhandari et al. analyzed the interobserver agreement among reviewers in categorizing the type of study, level of evidence, and subclassification for different clinical studies. The authors had 6 different surgeons with different levels of training in epidemiology analyzed 51 separate papers published in JBJS. The results demonstrated that the interobserver absolute agreement for the type of study and the level evidence was 82% and 67%, respectively. The epidemiology-trained reviewers had nearly perfect agreement in categorizing the type of study, level of evidence, and subclassification.

     

    Wright et al. published an editorial introducing the different types of study designs and defined the different levels of evidence. Illustration A is a figure from this editorial.

     

    Incorrect Answers:

    Answers 1, 3, 4, 5: The lowest quality study used in the systematic review determines the level of evidence.







     

    A 55-year-old male, alcoholic, presents to the ER with acute right knee swelling and pain x 3 days. He admits to prior episodes of this pain that resolve after a few days. Serum labs reveal an ESR of 40 mm/hr and CRP of 5 mg/dl. He undergoes right knee aspiration and based on the results, he is discharged home on colchicine with the presumed diagnosis of gout. Which of the following images of the aspiration results are consistent with this diagnosis?

    conclude that the patient populations and outcomes measure are homogenous and you do not have any concerns with randomization. You notice one of

    the studies included had 70% follow-up, yet the remaining studies had

    >80% follow-up. Knowing this, you appropriately assign what level of evidence to the systematic review?


    Level I Level II Level III Level IV Level V

    Corrent answer: 3

    The correct sequence of events should be the surgeon reads the surgical information on the consent to the patient, then the surgeon marks the surgical site with the patient’s assistance, then allows the anesthesia team to perform their procedure, and then performs a final Time-Out with the surgical team immediately prior to the surgical incision.

     

    Orthopedic surgical patients are at risk of surgical errors due the number of procedures that can be performed on the bilateral extremities. The responsibility to identify the correct surgical procedure at the correct location has expanded beyond only the surgeon. The entire surgical team is

    responsible for confirming the patient, surgical site, and surgical procedure. All members on the surgical team should be valued and emboldened to “speak up’ and actively participate. To help improve communication and reduce complications, surgical safety checklists have become common. In a statement

    published by the AAOS is 2015, they support the use of standardized surgical systems, including the use checklists, as it is critically important to keep patients safe. In 1998, the AAOS introduced the “Sign Your Site” safety program to reduce wrong-site surgeries through improved site identification. Permanent ink should be used to mark the site(s) with the patient's assistance prior to surgery, and the site(s) should be confirmed by the surgical team during the Time-Out immediately before the start of the surgical procedure.

     

    Singer et al. performed a study to evaluate the association between surgical teamwork and surgery safety checklist performance. Their results emphasized the importance of surgeon buy-in and clinical leadership to initiating and maintaining surgical safety checklists. In addition to surgeon buy-in and clinical leadership, factors that help maintain high-quality and consistent surgical teamwork were communication, coordination, respect, and assertiveness.

     

    Incorrect Answers:

    Answers 1, 2, 4, 5: These sequences of events are not recommended to confirm the patient, surgical site, and procedure accurately and effectively.





     

    You are reviewing a systematic review on the 90-day complication rate and outcome for same day total joint arthroplasty for publication. After you analyze the methodology of the 6 randomized controlled trials included in the review, you

    preoperative paperwork outside the room. The patient is taken to surgery and receives an interscalene block on the left shoulder after sedation. At the final Time- Out, the surgeon realizes a discrepancy with the laterality when the consent is read aloud. The surgeon aborts the case and wakes the patient. What is the correct sequence of events that should have happened to prevent this error? A: The surgeon begins

    the surgery B: The surgical team performs a Time-Out C: The surgeon marks the surgical site D: The surgeon reads the surgical information on the consent to the patient E: The anesthesia team administers a local extremity block


    E, C, B, D, A D, B, C, E, A D, C, E, B, A C, D, E, B, A D, E, B, C, A

    Corrent answer: 3

     

    Enchondral ossification occurs with relative stability constructs, which is represented by the bridge plate in figure C.

     

    Fracture healing is a complex process that occurs in several key steps. The type of healing that occurs is dependent on the stability and strain of the fracture environment. In constructs with very little strain, also referred to as absolute stability, there is primary bone healing through Haversian remodeling. This produces very little callus and does not rely on a cartilage precursor. Relative stability constructs with higher strains produce a cartilage precursor, which subsequently ossifies in later stages of healing, also referred to as enchondral ossification.

     

    Perren reviewed the biological mechanisms of fracture healing. The author discussed the importance of skeletal stiffness for limb function in addition to the healing process that utilizes soft tissue compensatory mechanisms to aid

    in fracture healing. The author concluded that the goal of fracture healing is to obtain a functional limb to allow for daily mobility and activity.

     

    Gerstenfeld et al. investigated the effect of non-selective and COX-2 selective NSAIDs effects on bone healing in a rat model. They reported a significantly higher nonunion rate in valdecoxib treated rats compared to the ketorolac group. They also noted that withdrawal of either drug at six days resulted in prostaglandin E2 levels returning to normal levels after 14 days. The authors concluded that COX-2 specific NSAIDs inhibited bone healing greater than nonspecific NSAIDs with the magnitude of the effect dependent on the duration of treatment, but the effects on prostaglandin E2 levels appear reversible with discontinuation of the drug.

     

    Figure A is the AP radiograph of the left distal tibia with three lag screws through a spiral fracture. Figure B is the lateral radiograph of the right elbow with an olecranon plate.

    Figure C is the AP radiograph of the right distal femur with a lateral bridge plate. Figure D is an AP radiograph of the left ankle with a lag screw and neutralization plate on the distal fibula. Figure E is the lateral radiograph of the forearm with a compression plate on the radius.

     

    Incorrect Answers:

    Answers 1, 2, 4, and 5: All of these fixation methods would provide absolute stability and result in primary bone healing through Haversian remodeling.







     

    A 31-year-old man is scheduled to undergo a right shoulder arthroscopic labral repair. The surgeon is running behind and hurries to the preoperative holding area. The surgeon greets the patient and verbally confirms the operative site with the patient. The surgeon leaves the patient’s room and completes the appropriate



    Figure A Figure B Figure C Figure D Figure E
     

    Corrent answer: 4

     

    The yield point is the transition point between elastic and plastic deformation. The yield strength is defined as the amount of stress necessary to produce a specific amount of permanent deformation.

     

    Stress is the amount of force applied to a material and strain is the deformation resulting

    from that stress. This is graphically depicted as a stress- strain curve, where the X-axis represents strain and the Y-axis represents stress. The elastic modulus of a material is the linear region of the graph (rise over run/stress on strain). Remember, an elastic material is one that resists a change in shape (less strain or deformation under increasing stress). Non- linear regions include the toe region for some materials (tendons/ligaments) and the plastic zone, which occurs after the yield point.

     

    Mantripragada et al. provide a review of recent advances in designing orthopaedic implants. Of note, they discuss modifications to metallic implants to reduce unwanted effects, such as nickel-free stainless steel. They also go over newer alloys with desirable mechanical and biological properties, such as tantalum, niobium, zirconium, and magnesium.

     

    Kennedy et al. provide a classic in-vitro tension study of the human knee ligaments. They used an Instron Tension Analyzer to test the ultimate failure of the medial collateral, lateral collateral, anterior cruciate, and posterior cruciate ligaments at different loading rates. They found that the posterior cruciate ligament was the strongest (the other ligaments were all of

    comparable strength) and that microscopic failure occurred before macroscopic failure. Illustration A represents a stress-strain curve.

    Incorrect Answers:

    Answer 1: The necking region occurs prior to failure/fracture of a material and

    is a phenomenon especially associated with a ductile material; the diameter of the material is diminished prior to fracture.

    Answer 2: The ultimate strength is defined as the maximum stress that a solid material can withstand before failure.

    Answer 3: Toughness is defined as the amount of energy per volume a

    material can absorb before fracture and is the area under the stress-strain curve. Answer 5: The toe region is seen in materials such as ligaments and tendons and represents the straightening of the crimped ligament fibers.









     

    Which of the following fixation constructs would achieve fracture healing through enchondral ossification?

    Necking region Ultimate strength Toughness Yield Point Toe region
     

    Corrent answer: 5

     

    The preosteoclast (precursor to the osteoclast) is the only cell of myeloid origin. The remainder of the cells involved in bone formation, remodeling, and metabolism are of mesenchymal origin.

     

    Osteoclast signaling, function, and biology have grown increasingly well understood over the past few decades. Osteoclast activity is regulated by

    osteoblasts, thereby coupling bone formation and resorption. Osteoclast differentiation from myeloid precursor cells is stimulated by key molecules including RANK, PU-1, and CSF-1. An understanding of these molecular pathways is essential to developing effective directed anti-resorptive therapies.

     

    Zaidi et al. present a comprehensive review of proliferation, differentiation, and hormonal regulation of cells of the bone. The authors specifically discuss the unique origin of the osteoclast from the myeloid lineage and conversely the mesenchymal origin of the osteoblast. Furthermore, they highlight the

    most recent understanding of the molecular mechanisms involved in osteoclast formation

    and signaling, including M-CSF and RANKL.

     

    Caterson et al. discusses mesenchymal differentiation in the context of musculoskeletal regeneration. The authors review the growth factors and bioactive signaling molecules involved in directed differentiation itno the various mesodermal lineages including bone, cartilage, muscle, tendon, marrow, and adipose. They emphasize the importance of understanding these pathways to regenerative medicine.

     

    Illustration A is a diagram illustrating the difference between mesenchymal and myeloid lineages.

     

    Incorrect answers:

    Answer 1-4: All of these cells are of shared mesenchymal origin.









     

    The point on a stress-strain curve that separates the plastic and elastic regions is defined as which of the following:


    Chondroblast Osteoblast Osteoprogenitor cell Osteocyte Preosteoclast
     

    Corrent answer: 2

     

    Due to the risk of inducible clindamycin resistance in erythromycin-resistant MRSA, a D-test should be performed.

     

    Isolates of MRSA that are resistant to erythromycin have been shown to become resistant to clindamycin through a process called inducible resistance, which is conferred by a plasmid that alters the 50S ribosome binding site for both clindamycin and erythromycin. Thus, when culture results reveal erythromycin-resistant MRSA, a D-zone test should be performed to check for inducible clindamycin resistance. The D-zone test is performed by

    placing an erythromycin disk in proximity to a clindamycin disk on an agar plate inoculated with methicillin-resistant S aureus (MRSA). A zone of inhibition in the shape of the letter "D" is seen with an inducible strain and is considered a positive test. If the D- zone test is positive, then clindamycin should not be used because the strain of MRSA can become resistant to the treatment.

     

    Marcotte et al. published a review on community-acquired methicillin-resistant Staphylococcus aureus (CA-MRSA). They reported that clindamycin has activity against Streptococcus species, but it is not as predictable against CA- MRSA. Clindamycin also presents a risk for the development of Clostridium difficile colitis and inducible clindamycin resistance. for which a D-zone test should be performed when culture results reveal erythromycin resistance.

     

    Steward et al. performed a study to determine the efficacy of testing for induced clindamycin resistance in erythromycin-resistant Staphylococcus aureus. They reported that resistance to erythromycin and clindamycin can occur through methylation of their ribosomal target site (25), which is typically mediated by erm genes. They found that disk diffusion is the preferred method for testing S. aureus isolates for inducible clindamycin resistance.

     

    Illustration A is an image of a positive D-zone test, which indicates inducible clindamycin resistance.

     

    Incorrect Answers:

    Answer 1: An oxidase test is used to classify bacteria as oxidase-positive or oxidase- negative and is based on whether or not a bacterium produces certain cytochrome c oxidases.

    Answer 3: A CAMP test is used to identify group B β-hemolytic streptococci. Answer 4: A Ziehl–Neelsen stain is used to identify acid-fast organisms such as mycobacteria.

    Answer 5: An Etest is used to determine the MIC (Minimum Inhibitory Concentration) and whether or not a specific strain of bacterium is susceptible to the action of a specific antibiotic.







     

    Which of the following cells involved in bone metabolism derives from a myeloid origin?

    Oxidase test D-zone test CAMP test Ziehl–Neelsen stain Etest
     

    Corrent answer: 3

     

    Enoxaparin primarily exerts its effects by inhibiting Factor Xa, which is labeled C in Figure A.

     

    Enoxaparin is a low molecular weight heparin (LMWH) that primarily exerts its effects by inhibiting Factor Xa. It achieves this by binding to antithrombin to form a complex that irreversibly inactivates clotting factor Xa. Enoxaparin has the advantage of not requiring laboratory monitoring and can be reversed with protamine sulfate. However, it is important to note that protamine sulfate is less effective in reversing enoxaparin compared to unfractionated heparin (UFH).

     

    Hyers published a review on the past, present, and future management of venous thromboembolism. He found that, for the most part, LMWH and other newer anticoagulants have been shown to be superior to UFH in terms of the venographic endpoint. He also reports that several meta-analyses have demonstrated that LMWH offers superior benefit to UFH for VTE prevention in hip and knee surgery patients.

    Tørholm et al. performed a study to determine outcomes of thromboprophylaxis using LMWH compared to placebo in elective hip surgery. They found that 9 (16%) patients in the treatment group and 19 (35%) in the placebo group developed deep venous thrombosis. The risk of thrombosis in the placebo group was increased with prolonged surgery and occurred more frequently during the first 4 postoperative days. They concluded that LMWH offers safe and easily administered thromboprophylaxis in total hip replacement.

     

    Figure  A  is an image  of  the coagulation  cascade.  Illustration A is an image     of  the

    coagulation cascade with the sites of action of the various anticoagulants labeled.

     

    Incorrect Answers:

    Answers 1, 2, and 4: Warfarin acts indirectly on Factors II, VIIa, IX, and X by inhibiting vitamin K 2,3-epoxide reductase.

    Answer 5: Dabigatran and bivalirudin are direct thrombin inhibitors.









     

    A 42-year-old IV drug user presents to the emergency department with a large abscess on his forearm. A bedside I&D is performed and he is started on broad-spectrum IV antibiotics. Initial results from his cultures demonstrate methicillin-resistant Staphlycoccus aureus (MRSA) that is also resistant to erythromycin. The team would like to transition him to oral clindamycin. Prior to transitioning him to clindamycin, which additional laboratory test should be performed?



    A B C D E
     

    Corrent answer: 5

     

    Teriparatide promotes bone formation in patients at high risk of fractures due to severe osteoporosis that is refractory to multiple treatments, including bisphosphonates and cement augmentation. Teriparatide is a human recombinant N-terminal parathyroid hormone.

     

    Teriparatide administered in daily injections results in bony formation, whereas continuous infusion results in bony resorption. In rat models, teriparatide caused an increase in the incidence of osteosarcoma, and thus should only be prescribed for patients for whom the potential benefits outweigh the potential risk. It can be administered in isolation or as an adjunct treatment during bisphosphonate therapy. However, in patients on long-term bisphosphonate therapy, discontinuation of bisphosphonates are advised to reduce potential complications of atypical femur fractures and jaw osteonecrosis.

    Watts et al. published a review article on postmenopausal osteoporosis. They reported that bisphosphonates can accumulate in bone, thus after a period of treatment, lower- risk patients should be offered a drug holiday. Denosumab, on the other hand, is not sustained when treatment is discontinued, so no drug holiday is warranted. They concluded that, although there are safety

    concerns regarding atypical femoral fracture and osteonecrosis of the jaw with long term use, the benefit of hip fracture risk reduction far outweighs the risk of these relatively uncommon side effects.

     

    Song et al. performed a meta-analysis to investigate the effect of teriparatide monotherapy and the additive effect of teriparatide on antiresorptive agents in postmenopausal women with osteoporosis. They reported that teriparatide monotherapy significantly improved bone mineral density (BMD) in the lumbar spine, total hip, and femoral neck compared with placebo; the additive effect

    of teriparatide over hormone replacement therapy (HRT) and denosumab agents was evident in all 3 skeletal sites; however, teriparatide plus alendronate did not demonstrate additive effect in total hip and femoral neck. They concluded that, for patients with osteoporosis who were at high risk for fracture, BMD increased more in patients receiving teriparatide than in those receiving alendronate.

     

    Saag et al. compared the use of teriparatide or alendronate in the management of glucocorticoid-induced osteoporosis. They reported that BMD had increased more in the teriparatide group than in the alendronate group in the lumbar spine and total hip at 6 and 12 months, respectively. They also reported significantly fewer new vertebral fractures in the teriparatide group compared to the alendronate group. They concluded that in severely osteoporotic patients at high risk for fracture, BMD increased more in patients receiving teriparatide than in those receiving alendronate.

     

    Figure A depicts multiple vertebral insufficiency fractures in the setting of a prior cement augmentation procedure.

     

    Incorrect Answers:

    Answer 1 & 4: While calcium and vitamin D supplementation is a good prophylactic agent in all patients, jaw osteonecrosis is not a described risk. Answers 2 & 3: Long term use of non-nitrogen (produces toxic adenosine triphosphate analog) and nitrogen-containing (inhibits farnesyl pyrophosphate synthase) bisphosphonates are not recommended due to the risk of jaw osteonecrosis and atypical femur fractures.




     

    Where in the coagulation cascade shown in Figure A does enoxaparin primarily exert its effects?



    Calcium supplementation Produces toxic adenosine triphosphate analog Inhibits farnesyl pyrophosphate synthase Vitamin D supplementation Recombinant N-terminal parathyroid hormone

    Corrent answer: 3

    This patient is presenting with signs of a septic nonunion after open reduction and internal fixation (ORIF) of a radial shaft fracture. Of the choices listed, C- reactive protein (CRP) is the best predictor of infection in the setting of nonunion.

     

    Nonunions after fracture fixation may occur from infection. The most sensitive and readily-available laboratory marker to detect infection is the CRP. CRP is an acute phase reactant that significantly rises within 6 hours after tissue damage or onset of clinical infection. CRP then peaks 2-3 days later and returns to normal levels 5-21 days after the inciting event if it is treated (e.g. antibiotics for cellulitis). In septic nonunions, the chance of fracture healing is low if the infection is not properly treated, and chronic infection can lead to substantially elevated CRP values.

     

    Wang et al. evaluated the effectiveness of laboratory tests in the diagnosis of

    infected nonunion. They reported that the sensitivity and specificity of CRP for detection of infected nonunions are both higher than those of IL-6. They concluded that the diagnostic utility of CRP was superior to IL-6, which is contrary to similar studies comparing these markers in prosthetic joint infection patients.

     

    Stucken et al. performed a study to investigate the utility of a standardized protocol to rule out infection in high-risk patients and to evaluate the efficacy of each component of the protocol. They reported that the ESR and the CRP levels were both independently accurate predictors of infection. They

    concluded that their protocol can help surgeons to risk-stratify patients prior to the surgical treatment of a nonunion, allowing them to counsel patients more appropriately.

    Figure A depicts a nonunion of a radial shaft fracture after ORIF. Incorrect Answers:

    Answers 1, 2, 4, 5 : Interleukin-1, 6, WBC count with differential, and ESR are not as sensitive and specific as CRP in diagnosing infection in setting of nonunion.





    FOR ALL MCQS CLICK THE LINK ORTHO MCQ BANK

     

    An 85-year-old woman presents with severe back pain and the CT shown in Figure A. Her history is notable for prior vertebral compression fractures for which she underwent a cement augmentation procedure. She has been on bisphosphonates for the last 5 years, without improvement of her osteoporosis. She has no history of malignancy. What is the mechanism of action of the medication that should be prescribed for her refractory osteoporosis?

    Interleukin-1 White blood cell (WBC) count with differential C-Reactive Protein (CRP) Interleukin-6 Erythrocyte sedimentation rate (ESR)
     

    Corrent answer: 1

     

    A receiver operating characteristic (ROC) curve is used to determine responsiveness.

     

    Responsiveness is a measure of the diagnostic ability of different tests. It can be determined by calculating the C-statistic, which represents the area under a

    Receiver Operating Characteristic (ROC) curve. On a ROC curve, the false positive rate (1 - specificity) is plotted on the x-axis, while the true positive rate (sensitivity) is plotted on the y-axis. The higher the area under the curve, the more responsive the outcome measure. A value of 0.5 indicates a random chance and a therefore useless test, while values above 0.75 usually are considered to be adequately responsive.

    Kocher et al. published a review on clinical epidemiology and biostatistics for orthopaedic surgeons. They reported that the relationship between the sensitivity and specificity of a diagnostic test can be portrayed with use of a receiver operating characteristic (ROC) curve. A ROC graph shows the relationship between the true- positive rate (sensitivity) on the y-axis and the false-positive rate (1 − specificity) on the x-axis plotted at each possible cutoff. Overall diagnostic performance can be evaluated on the basis of the area under the ROC curve.

     

    Hanley et al. published a review on the meaning and use of the area under a receiver operating characteristic (ROC) curve. They reported that it represents the probability that a randomly chosen diseased subject is (correctly) rated or ranked with greater suspicion than a randomly chosen non-diseased subject.

     

    Illustration A is an example of a ROC curve. Illustration B is an example of a funnel plot. Illustration C is an example of a Kaplan-Meier curve. Illustration D is a table outlining the interpretation of the Cronbach alpha coefficient. Illustration E is an example of a forest plot.

     

    Incorrect Answers:

    Answer 2: A funnel plot is used to evaluate for the presence of publication bias. Answer 3: A Kaplan–Meier curve is used to determine the survivorship of a population.

    Answer 4: The Cronbach alpha coefficient is used to measure internal consistency and indicates how well individual items in a test or questionnaire are correlated.

    Answer 5: A forest plot is used to graphically depict a meta-analysis of the results of randomized controlled trials.








     

    A 32-year-old man underwent open reduction and internal fixation for an open radial shaft fracture 6 months ago. He is now experiencing fevers and chills at night and pain and swelling over the surgical site. A current radiograph is depicted in Figure A. What is the most accurate laboratory test for assessing his most likely diagnosis?


    Receiver Operating Characteristic (ROC) curve Funnel plot Kaplan–Meier curve Cronbach alpha coefficient Forest plot
     

    Corrent answer: 1

     

    The Patient-Reported Outcomes Measurement Information System (PROMIS) has been shown to have reduced floor and ceiling effects compared to other assessment tools.

     

    The PROMIS system was developed to produce a highly reliable, precise, and versatile assessment of outcomes. When administered in a computerized adaptive mode, each question that is answered is followed with a customized follow-up question based on the previous response, which allows for reduced testing items and time. Further, the results of the assessment are reported in T-scores with 50 being the population norm and with a standard deviation of

    10. This reporting avoids a ceiling and floor effect, where most of the respondents score either at the top or the bottom of the assessment scale.

     

    Brodke et al. performed a comparative study of the psychometric properties of

    the PROMIS physical function domain, the Oswestry Disability Index (ODI), and the Short Form 36 (SF-36) questionnaire in the spine population. They reported that the PROMIS PF CAT required less time and had less ceiling and floor effects than the ODI and SF-36 questionnaires. Additionally, there was a high correlation between the three evaluation methods. They concluded that the PROMIS system provides better outcomes assessment while taking less time and fewer questions than legacy tests.

     

    Brodke et al. reviewed the literature regarding the use of the PROMIS outcomes measure for several orthopedic conditions. The use of the computerized adaptive testing version allows for a higher level of precision with fewer questions administered and reduced time. In the spine literature, the PROMIS system has shown a reduced ceiling and floor effect compared to other legacy tests. They concluded that this system provides a highly reliable and valid means of measuring patient-reported outcomes while reducing respondent and administrative burden.

     

    Incorrect Answers:

    Answer 2: The use of computerized adaptive testing has reduced the number of questions required for assessment without compromising precision.

    Answer 3: Several studies have reported a high correlation of the PROMIS system with the Oswestry Disability Index and Short Form 36.

    Answers 4 and 5: The PROMIS system provides a highly reliable and valid assessment of patient-reported outcomes.







     

    Which of the following statistical tools is used to determine responsiveness?


    Reduced floor and ceiling effect Increased question items Poor correlation with other outcomes assessment tools Low reliability Low validity
     

    Corrent answer: 2

     

    The only accurate statement is that, at 1 month, patients in group 3 are more likely to live longer than patients in group 1 and 2.

     

    The Kaplan-Meier curve is a statistical method used in survivorship analysis to estimate survival rates at different time points. The horizontal axis depicts the time points and the vertical axis depicts the probability of survival. At a given time point, greater survivorship is demonstrated by higher values on the vertical axis.

     

    Kocher et al. reviewed clinical epidemiology and biostatistics relevant in orthopaedic

    surgery. They reported that survivorship data are typically analyzed with use of the Kaplan-Meier method. This analysis is used when the actual date of the end point is known. A survivorship curve can be plotted to illustrate the percentage of patients still alive after the intervention.

     

    Petrie et al. reviewed statistics, terminology and the principles involved in simple data analysis, and outlines areas of medical statistics that have gained

    prominence in recent years in orthopaedic papers. The authors provide an insight into some of the more common errors that occur in published orthopaedic journals and which are frequently encountered at the review stage in papers submitted to the Journal of Bone and Joint Surgery.

     

    Figure A depicts a Kaplan-Meier curve for 3 groups of patients undergoing 3 different interventions.

     

    Incorrect Answers:

    Answers 1, 3, & 4: At 2 weeks and 1 month, patients in group 3 are more likely to live longer than patients in group 1 and 2 based on this Kaplan-Meier analysis.

    Answer 5: Cost-effectiveness cannot be determined with the limited information given.





     

    When evaluating patient-reported outcomes in spine surgery, the Patient- Reported Outcomes Measurement Information System (PROMIS) has been found to have what differences compared to other assessment tools?



    At 1 month, patients in group 1 are more likely to be alive than patients in group 2 and 3 At 1 month, patients in group 3 are more likely to be alive than patients in group 1 and 2 At 2 weeks, patients in group 1 are more likely to be alive than patients in group 2 and 3 At 2 weeks, patients in group 2 are more likely to be alive than patients in group 1 and 3 The intervention rendered on patients in group 1 is the most cost-effective
     

    Corrent answer: 2

     

    Communication failures have been associated with increased treatment costs, increased complications, and delays in patient care.

    The leading cause of medical errors, wrong-site surgeries, diagnostic delays and loss to follow-up is attributed to communication errors. Studies have shown that this directly leads to increased complications, increased treatment costs, and delays in patient care. In the surgical setting, the performance of the pre-surgical time-out affords clarification of the treatment plan, confirmation of the procedure and site, and an opportunity for the team to address any concerns.

     

    Weller et al. reviewed barriers to effective teamwork in healthcare delivery. They found that effective communication across all team members, inclusive training, and the creation of democratic teams can reduce treatment delays and minimize complications. They concluded that all team members should have a shared understanding of the situation when involved in patients care.

     

    Lingard et al. performed a prospective study determining the feasibility of a preoperative 10-item checklist discussion including nurses, surgeons, and anesthesiologists. They reported that the discussion usually took place prior to bringing the patient to the OR and took on average 3.5 minuted. Overall, the guided discussion was very well received by participants and provided an opportunity to inform providers of the patient's information to address potential problems. The authors cited that variations in workflow patterns as a potential barrier to widescale implementation of the checklist discussion.

    Incorrect answers:

    Answers 1, and 3-5: Communication failures have been associated with increased treatment costs and increased complications.





     

    The outcomes of three novel interventions are investigated and the preliminary results are depicted in Figure A. Which of the following statements is most accurate?


    Decreased treatment costs and increased complications Increased treatment cost and increased complications Decreased treatment costs and complications Increased treatment costs and decreased complications No differences in complications and treatment costs
     

    Corrent answer: 4

     

    Linezolid acts to inhibit protein synthesis by preventing the formation of the initiation complex between the 30S and 50S subunits of the ribosome.

     

    Linezolid is a type of oxazolidinone, a relatively newer class of antibiotics that are active against multidrug-resistant staphylococci, streptococci, and enterococci. Although many antimicrobial agents affect or disrupt protein synthesis in bacteria, linezolid is unique in that it disrupts protein synthesis at its origin rather than later in the cascade. It achieves this by binding to the

    50S subunit of the bacterial ribosome, preventing the formation of the initiation complex.

     

    Swaney et al. performed a study to investigate the mechanism of action of linezolid. They found that linezolid inhibits the formation of the initiation complex in bacterial translation systems by preventing the formation of the N- formylmethionyl-tRNA-ribosome-mRNA ternary complex.

     

    Thompson et al. performed a study on the efficacy of various antibiotics in treating periprosthetic joint infections (PJI). They administered oral linezolid with or without oral rifampin, intravenous vancomycin with oral rifampin, intravenous daptomycin or ceftaroline with or without oral rifampin, oral doxycycline, or sham treatment at human- exposure doses for 6 weeks in a mouse model of PJI. They found that oral-only linezolid-rifampin and all intravenous antibiotic-rifampin combinations resulted in no recoverable bacteria and minimized reactive bone changes. Additionally, although oral linezolid was found to be the most effective monotherapy, all oral and intravenous antibiotic monotherapies failed to clear infection or prevent reactive bone changes.

    They concluded that the oral-only linezolid-rifampin option might reduce venous access complications and health-care costs.

     

    Illustration A is a diagram depicting the mechanism of action of various antibiotics.

    Incorrect Answers:

    Answer 1: Gentamicin is an aminoglycoside that acts to inhibit protein synthesis by binding to the 30S subunit of the bacterial ribosome, impairing the ability of the ribosome to discriminate proper transfer RNA and messenger RNA interactions.

    Answer 2: Doxycycline is a tetracycline that acts to inhibit protein synthesis by binding to the 30S subunit of the bacterial ribosome, preventing the binding of transfer RNA to messenger RNA.

    Answer 3: Chloramphenicol acts to inhibit protein synthesis by binding to the

    50S subunit of the bacterial ribosome, preventing protein chain elongation by inhibiting peptidyl transferase activity of the ribosome.

    Answer 5: Erythromycin is a macrolide that acts to inhibit protein synthesis by binding to the 50S subunit of the bacterial ribosome, preventing protein chain

    elongation by blocking aminoacyl translocation.







     

    Failure of effective communication in the surgical setting has been associated with what?


    Gentamicin Doxycycline Chloramphenicol Linezolid Erythromycin
     

    Corrent answer: 5

     

    Unfractionated heparin works in the coagulation cascade by promoting the ability of antithrombin III to inhibit factors IIa, III, Xa.

     

    Heparin works by binding to and enhancing the ability of antithrombin III to inhibit factors IIa, III, Xa. It is metabolized by the liver. The risks associated with the use of unfractionated heparin include bleeding and heparin induced thrombocytopenia (HIT). The reversal agent is protamine sulfate.

    Agnelli et al. reviewed direct thrombin inhibitors for the prevention of VTE after major orthopaedic surgery. They reported widespread use, however, limitations in the use of unfractionated heparin and low-molecular-weight heparins. They highlighted that direct thrombin inhibitors (Ximelagatran) inactivate thrombin without requiring any plasma cofactor, inhibit both free

    and fibrin-bound thrombin, and do not appreciably bind to plasma proteins. They concluded that the features of rapid absorption, conversion, bioavailability, low variability, dose-time, food independency, and the predictable anticoagulant activity make direct thrombin inhibitors an attractive antithrombotic agent especially for prolonged out-of hospital prophylaxis.

     

    Kwong et al. reviewed the efficacy and safety of fondaparinux, a selective factor Xa inhibitor and reported its efficacy and safety in 4 phase III clinical trials. They reported fondaparinux usage resulted in an overall 55% decrease in the risk of venous thromboembolism (VTE) relative to the low-molecular- weight heparin enoxaparin without increasing the incidence of clinically relevant bleeding, which was similarly low for both agents. They concluded

    that the superior efficacy of fondaparinux relative to enoxaparin is the result of its unique mechanism of action, clinical pharmacology.

     

    McLynn et al. investigated the risk factors and chemoprophylaxis for VTE in elective spine surgery. They reported that independent risk factors for VTE included greater age, male gender, increasing body mass index, dependent functional status, lumbar spine surgery, longer operative time, perioperative blood transfusion, longer length of stay, and other postoperative

    complications. The majority of patients received unfractioned heparin and they observed this did not significantly influence the rate of VTE, but was associated with a significant increase in hematoma requiring a return to the operating room. They concluded that there is insufficient evidence to support the routine use of chemoprophylaxis in low-risk patients, and recommended full consideration of risks and benefits after elective spine surgery.

     

    Figure A depicts the coagulation cascade, with the factor circled in red (IIa) being the target of unfractionated heparin, through its effect on antithrombin III.

     

    Incorrect Answers:

    Answer 1: Aspirin irreversibly blocks the formation of thromboxane A2 in platelets, which inhibits platelet aggregation and thus the formation of a clot. Answer 2: Fondaparinux is an indirect factor Xa inhibitor that works through antithrombin III. Unlike unfractionated heparin, it is selective for factor X and does not have effect on factor IIa.

    Answer 3: Rivaroxaban has a direct effect on factor Xa inhibition, not factor IIa.

    Answer 4: TXA competitively inhibits the activation of plasminogen.




     

    Which of the following antibiotics acts to inhibit protein synthesis by preventing the formation of the initiation complex between the 30S and 50S subunits of the ribosome?





    Aspirin Fondaparinux Rivaroxaban Tranexamic acid (TXA) Heparin
     

    Corrent answer: 1

     

    Acetaminophen inhibits prostaglandin E2 production through its effect on interleukin-1 beta. It exerts its effects on the central nervous system.

     

    Multimodal pain management has become an increasingly important consideration in total joint arthroplasty recovery, namely in regard to minimizing opioid requirements while optimizing pain control and patient satisfaction. Given the negative side effects and increasing scrutiny on narcotic use, modalities that help to decrease post-operative narcotic requirements have become a major focus and include nonsteroidal anti- inflammatories, selective COX-2 inhibitors, gabapentinoids, local anesthetic blocks, and acetaminophen. Acetaminophen acts directly the central nervous system, providing both pain relief and antipyretic effects. It does so by inhibiting prostaglandin E2 production through the blockade of interleukin-1 beta.

     

    Jinnah et al. performed a comparison of two multimodal pain regimens used for postoperative pain control in total joint arthroplasty patients. They hypothesized that using a multimodal pain protocol focusing on periarticular injections including liposomal bupivacaine would have improved results when compared with a parenteral opioid- based regimen. They found a decrease in length of stay and rate of discharge to skilled nursing facilities with the implementation of a novel multimodal protocol. They concluded that a multimodal approach inclusive of periarticular injection can lead to a reduced

    length of stay.

     

    Politi et al. performed a prospective randomized trial comparing the use of intravenous versus oral acetaminophen in total joint arthroplasty. They found that IV acetaminophen did not provide a significant difference in pain relief except within the first 0-4 hours. They concluded that IV acetaminophen does not provide a significant benefit when compared to the oral form, which is also less expensive.

    Incorrect Answers:

    Answer 2: Acetaminophen exerts its effects on the central nervous system. Answers 3, 4, & 5: Though it's mechanism is not fully understood, acetaminophen's effects on cyclooxygenase-1 or 2, are yet to be determined.





     

    Which of the following agents results in blood anticoagulation by exerting inhibitory effects on the factor circled in the color red in Figure A?


    Acetaminophen Prednisone Ibuprophen Morphine Gabapentin
     

    Corrent answer: 1

     

    Based on the stress-strain curve in Figure A, ceramic has the highest modulus of elasticity.

    Young's modulus is a measure of the stiffness (ability to resist deformation) of a material in the elastic zone that is calculated by measuring the slope of the stress-strain curve in the elastic zone. A higher modulus of elasticity indicates a stiffer material. Relative values of Young's modulus include: (1) ceramic, (2) alloy, (3) stainless steel, (4) titanium, (5)

    cortical bone, (6) matrix polymers, (7) PMMA, (8) polyethylene, (9) cancellous bone, (10) tendon/ligaments, and (11) cartilage.

     

    Lang et al. published a review on the use of ceramics in total hip replacement. They report that properties of ceramic which make it particularly attractive for this application include its hardness, high compression strength, and excellent wettability. However, its low fracture toughness and linear elastic behavior make it prone to breakage under stress.

     

    Figure A is a stress-strain curve, the slope of which is used to determine Young's modulus.

     

    Incorrect Answers:

    Answers 2, 3, 4, and 5: Ceramic has the highest modulus of elasticity.

     

    Which of the following medications work by inhibiting prostaglandin E2 production through interleukin-1 beta?



    Ceramic PMMA Titanium Cancellous bone Cartilage

    Corrent answer: 1

    A power analysis determines the minimum number of patients needed in a study to show a clinically significant difference. It should be performed before starting the study.

     

    Power is the probability of finding a significant association if one truly exists. It is defined as 1 minus the probability of a type 2 error (beta) and is generally set to 80% meaning that we are willing to accept a 20% chance of a type 2 error. A power analysis will determine the minimum number of patients that shows a clinically significant difference. The minimum number of patients should be determined prior to initiating a study. Calculation of power after a study has been completed is controversial and discouraged.

    Vavken et al. performed a review on the management of confounding in controlled orthopaedic trials. They report that confounding occurs when the effect of exposure of an outcome is distorted by a confounding factor and will lead to spurious effect estimates in clinical studies. They performed a cross- sectional study of a sample of controlled trials reported in the orthopaedic literature in 2006. They conclude that only 30 studies reported that the had a power analysis performed.

     

    Kocher et al. performed a review of clinical epidemiology and biostatistics as a primer for orthopedic surgeons. They report that a power analysis should be performed prior to initiating the study. This will ensure the study is appropriately powered and decrease the effects of chance. They conclude that when a study determines no significant effect the power of the study should be reported.

     

    Incorrect Answers:

    Answers 2, 3, 4, & 5: A power analysis determines the minimum number of patients that shows a clinically significant difference and should be performed prior to initiating the study.





     

    Based on the stress-strain curve in Figure A, which of the following materials has the highest modulus of elasticity?


    Determines the minimum number of patients that shows a clinically significant difference; prior to initiating the study Determines the minimum number of patients that shows a clinically significant difference; after the study's completion Determines the maximum number of patients that shows a clinically significant difference; prior to initiating the study Determines the maximum number of patients that shows a clinically significant difference; during the study Determines the maximum number of patients that shows a clinically significant difference; after the study's completion
     

    Corrent answer: 3

     

    The World Health Organization (WHO) defines osteopenia as a bone mineral density (BMD) that is between 1 and 2.5 standard deviations below young normal (T-score of - 1 to -2.5).

     

    The WHO defines osteopenia as decreased BMD without fracture risk. Osteopenia is defined as a T-score of 1 to 2.5 standard deviations below young normals (-1 to -2.5), while osteoporosis is defined as a T-score of more than

    2.5 standard deviations below young normals (< -2.5). The Z-score represents a comparison to age-matched normals.

    Johnell et al. published a study on the predictive value of BMD for hip and other fractures. They found that lower BMD was associated with a higher risk of osteoporotic fracture. They concluded that BMD is a risk factor for fracture of substantial importance and is similar in both sexes.

     

    Lee et al. published a study on T-Score discordance of BMD in patients with atypical femoral fractures (AFF). They found that the prevalence of T-score discordance between the hip and lumbar spine was relatively high in patients with AFF, and the presumed age at the initiation of bisphosphonate therapy was younger in patients with discordant T-scores.

     

    Incorrect Answers:

    Answer 1: The Z-score is the BMD relative to similarly aged patients, not to normal young matched controls.

    Answer 2: The T-score is the BMD relative to normal young matched controls, not to similarly aged patients.

    Answers 4 and 5: Osteopenia and osteoporosis are defined by the T-score, not the Z- score.







     

    You are planning on performing a study to determine if there is a difference in healing rates for distal femur fractures by placing patients on Vitamin D after injury. With regards to your study, the statistician mentions a power analysis. What is the purpose of power analysis and when should it be performed?


    The Z-score is the BMD relative to normal young matched controls The T-score is the BMD relative to similar aged patients Osteopenia is defined as a T-score of -1 to -2.5 Osteopenia is defined as a Z-score of -1 to -2.5 Osteoporosis is defined as a Z-score of < -2.5
     

    Corrent answer: 1

     

    This patient has an impending subtrochanteric femur fracture and should be stabilized with cephalomedullary nailing.

     

    Bisphosphonates have been shown to prevent osteoporotic fractures. They suppress osteoclastic recruitment and activity and induce apoptosis of osteoclasts. However, they have also been associated with subtrochanteric femur fractures. Cortical stress reactions in the form of lateral cortical thickening have been documented when radiographs were performed during the prodromal period preceding these fractures. If radiographs are obtained and demonstrate lateral cortical thickening in the presence of thigh pain, the entire femur should be stabilized with prophylactic cephalomedullary nailing to prevent fracture.

     

    Shane et al. performed a review of atypical subtrochanteric and diaphyseal

    femoral fractures. They report that long-term bisphosphonate use is associated with these injuries. Bisphosphonates localize in areas that are developing stress fractures and suppress intracortical remodeling. When bisphosphonate use has stopped, the risk of fracture decreases over time. They conclude that teriparatide may advance the healing of atypical femur fractures after surgical treatment.

     

    Koh et al. studied the natural history of femoral stress lesions associated with bisphosphonate therapy. They determined certain features that predispose to complete stress fractures. They found all patients had thigh pain before fracture. They conclude that cortical stress reactions associated with prolonged antiresorptive therapy and the "dreaded black line" should be prophylactically stabilized to avoid a complete fracture.

     

    Figure A is a radiograph of the proximal femur demonstrating lateral cortical thickening with the "dreaded black line." Illustration A is the same image with an arrow indicating the "dreaded black line."

     

    Incorrect Answers:

    Answers 2, & 4: Although this patient's smoking history is concerning, the radiographs are diagnostic. Oncologic workup including transfer to an orthopaedic oncologist is not necessary.

    Answer 3: Standard antegrade nailing would not protect the femoral neck in this pathologic fracture. The entire femur should be protected in this case. Answer 5: Observation is not recommended at this time as this patient is at high risk of a subtrochanteric femur fracture.







     

    Which of the following statements regarding bone mineral density (BMD) is true?



    Cephalomedullary nailing Computed tomography of the chest, abdomen, and pelvis Standard antegrade femoral nailing Transfer to an orthopaedic oncologist Observation
     

    Corrent answer: 4

     

    The 2017 American College of Rheumatology/American Association of Hip and Knee Surgeons Guideline for the Perioperative Management of Antirheumatic Medication states that hydroxychloroquine can be continued and etanercept

    should be held for 2 weeks prior to undergoing total hip arthroplasty.

     

    Patients with rheumatoid arthritis (RA) report high satisfaction following hip or knee replacement despite the higher rates of infection, dislocation, and readmission rates. Patients with RA may present on a variety of different biologic and nonbiologic medications used to control their systemic RA.

    Optimal preoperative management of these immunosuppressant medications may help mitigate some of the risks of postoperative complications in RA patients.

     

    Singh et al. reviewed the evidence surrounding the benefits and harms of various antirheumatic medications. They found evidence for traditional DMARDs, biologic agents, and nonbiologic agents in acute and established RA totaling 74 recommendations. They concluded that these recommendations are not prescriptions and that ultimate decision making should be patient- specific in a shared-decision

    making process between the patient and physician.

     

    Goodman et al. performed a multistep literature review on optimal antirheumatic medication management prior to joint replacement surgery. They were able to provide recommendations on when to continue, when to withhold, and when to restart these medications, and the optimal perioperative dosing of glucocorticoids. They concluded that these guidelines will help physicians manage antirheumatic medications at the time of elective THA or TKA.

     

    Incorrect Answers:

    Answers 1, 2, 3, and 5: These answers are not the correct combination of preoperative dosing for hydroxychloroquine and etanercept.





     

    An 83-year-old woman presents complaining of thigh pain. The pain has been progressing over the last few months. She denies any night chills or recent weight loss. She has smoked 1 pack per day for the last 40 years. Her current medications are alendronate and citalopram. Her current imaging is shown in Figure A. What is the next best step in treatment?


    Continue hydroxychloroquine and etanercept Hold hydroxychloroquine 1 week prior to surgery and continue etanercept Continue hydroxychloroquine and hold etanercept 1 week prior to surgery Continue hydroxychloroquine and hold etanercept 2 weeks prior to surgery Hold hydroxychloroquine and etanercept for 2 weeks prior to surgery

    Corrent answer: 4

     

    The patient is presenting with complex regional pain syndrome (CRPS) type 1 after a distal 1/3 tibial shaft fracture. The best diagnostic test for this is a thorough history and physical exam.

     

    CRPS is a disorder of increased sympathetic activity in a region of prior trauma. Cases can be classified as type 1, where there is no demonstrable nerve damage, or type 2, where a specific nerve is affected. Patients will typically present with cool and mottled skin that atrophic and absent of hair. Many times the affected limb will be noticeably cooler than the contralateral side. In advanced stages, there will be joint contractures and extensive osteopenia on radiographs. Several diagnostic aids have been developed, but remain inadequate to diagnostic sensitivity compared to a thorough history and physical.

     

    Shah et al. reviewed the diagnosis and treatment of CRPS. The authors reported that sweat quantification testing, skin thermography, and electromyography may be useful aids in diagnosis, but there is a lack of diagnostic sensitivity to make these tests reliable. The authors concluded that evidence suggests gabapentin, prazosin, propranolol, nifedipine, and mexiletine are the best medications for treatment.

     

    Hogan et al. reviewed the diagnosis and treatment of CRPS. The authors reported that the most sensitive means for diagnosis is a good history and physical exam as there is no single test to confirm the diagnosis. The authors recommended a multidisciplinary team approach including pain specialists, physical therapists, and orthopedic surgeons as syndrome response to medications is variable.

     

    Figures A and B demonstrate an AP and lateral radiograph of the right tibia and fibula with a distal 1/3 tibia fracture. Illustration A depicts that Lankford and Evans classification for CRPS.

     

    Incorrect Answers:

    Answer 1: The patient does not have compartment syndrome, which would best be addressed with compartment releases.

    Answer 2: Complex regional pain syndrome type 2 is defined by CRPS symptoms in a single nerve distribution. This patient is experiencing symptoms in the entire extremity which is consistent with CRPS type 1.

    Answer 3: The patient's history and physical exam are most consistent with CRPS and not popliteal artery aneurysm, which would be best assessed with an angiogram.

    Answer 5: Reflex sympathetic dystrophy, also known as CRPS, will not demonstrate significant findings on EMG testing.







     

    A 72-year-old female with rheumatoid arthritis is scheduled to undergo total hip arthroplasty. She presents for her preoperative visit and asks about dosing of her antirheumatic medications. She currently takes etanercept weekly and hydroxychloroquine daily. Which of the following is the best dosing recommendation for her antirheumatic medications prior to surgery?

    the entire right lower extremity, with sensitivity to cold temperatures. Physical exam demonstrates hyperesthesia of the extremity, thin and shiny appearing skin, cyanotic appearing with skin cool to the touch. What is the likely diagnosis and what is the best diagnostic test?


    Undiagnosed compartment syndrome; compartment pressure checks Complex regional pain syndrome type 2; bone scan Popliteal artery aneurysm; angiogram Complex regional pain syndrome type 1; history and physical exam Reflex sympathetic dystrophy; electromyography and nerve conduction velocities
     

    Corrent answer: 3

     

    In a 5-year-old female without a history of trauma or rashes and with persistent oligoarthritis that improves during the day, the most likely diagnosis is juvenile idiopathic arthritis (JIA). Early-onset JIA is associated with chronic uveitis.

    JIA is defined by the American College of Rheumatology as persistent arthritis and swelling in one or more joints for 6 weeks or longer in a patient younger than 16 years of age. It is a diagnosis of exclusion, usually entailing pattern recognition after a thorough history and physical exam. Serologic studies, including rheumatoid factor (RF), antinuclear antibody (ANA), and HLA-B27, may be helpful to rule out other etiologies (septic arthritis, systemic lupus erythematosus, rheumatic fever); however, these are neither sensitive nor specific. In patients with JIA, evaluation for possible uveitis by an ophthalmologist should be considered. Although this patient could have Lyme disease given the likely recent exposure to ticks during her camping trip, the lack of a rash, unresponsiveness to antibiotics, and polyarthritic nature make it less likely.

     

    Arvikar et al. compared clinical features of systemic autoimmune arthritides to those of Lyme arthritis. They found that patients with Lyme arthritis usually had a clinical picture of monoarticular knee arthritis, whereas patients with systemic autoimmune arthritis manifested with polyarthritis. They concluded that systemic autoimmune arthritis with or without a history of Lyme disease should be treated with disease-modifying antirheumatic drugs (DMARDs).

     

    Punaro et al. reviewed common rheumatologic conditions in children who may present to orthopaedic surgeons. For JIA, they reported a typical history of oligoarthritis for 6 weeks or more in a white female patient, with a peak onset between ages 1 and 3 years. Uveitis was typically chronic, bilateral, and asymptomatic. They concluded that while serologic tests were useful in

    excluding other diagnoses, they were less useful in confirming JIA.

     

    Incorrect Answers:

    Answer 1: Pericarditis, not cardiomegaly, is associated with JIA. Answer 2: Bilateral facial nerve palsy is associated with Lyme disease. Answers 4 and 5: There is no association of JIA with lower limb hemihypertrophy or meningitis.







     

    A 40-year-old patient sustains the injury in Figures A and B six months ago and underwent the appropriate fixation method. The patient is continuing to experience a tremendous amount of pain in

    returning from summer camp. She denies any antecedent trauma, fevers, or rashes. Antibiotics prescribed by her primary care doctor have provided no significant relief, but she reports feeling better at

    the end of the day. Labs reveal a negative rheumatoid factor. Which of the following is most commonly associated with her diagnosis?


    Cardiomegaly Facial nerve palsy Uveitis Lower limb hemihypertrophy Meningitis
     

    Corrent answer: 3

     

    The hardware shown in Figure A is a tension band plate. It is able to perform its function due to the Hueter-Volkmann Law.

     

    Bones undergo continuous remodeling and turnover which are sensitive to the surrounding mechanical environment. Bone remodeling is governed by Wolff’s law, while the mechanical influence on longitudinal bone growth is controlled by the Hueter– Volkmann law. Wolff’s law relates to the adaptation of bone to its mechanical environment, and involves bone apposition stimulated by intermittent increased stress and bone resorption following reduced intermittent stress. The Hueter–Volkmann law relates to immature bone growth suppression through sustained compressive loading and growth acceleration by reduced loading or distraction.

     

    Villemure et al. performed a review of growth plate mechanics and mechanobiology. They report that growth plates are sensitive to the surrounding mechanical environment. There are a number of clinical conditions of the skeleton that are thought to result from abnormal mechanical loading conditions influencing longitudinal growth prior to skeletal maturity, such as clubfoot (associated with limb position in utero), slipped capital femoral epiphysis, tibia vara, spondylolisthesis, and scoliosis.

    Shabtai et al. performed a review of the limits of growth modulation using tension band plates in the lower extremities. Tension band plates have been found to be safe and effective at correcting pediatric frontal plane angular deformities. They found that the success rate for idiopathic cases nears 100%. The success rate for pathologic cases is lower and they have a higher complication rate. They conclude that tension band plates are a reasonable option for all but the most extreme frontal and sagittal plane deformities.

     

    Figure A is a bilateral knee radiograph of a pediatric patient with tension band plates on the right tibia. Illustration A is a bilateral knee radiograph of the same pediatric patient.

    The physis appears to have partially closed down and the angle of the screws has changed. One of the screws has broken which happens frequently.

     

    Incorrect Answers:

    Answer 1: Cutting cones are produced by osteoclastic tunneling. This is how cortical bone remodels.

    Answer 2: Wolff's Law states that bone remodels in response to mechanical stress. Answer 4: The piezoelectric effect occurs when bone remodels in response to electric charges.

    Answer 5: Compression plating is a technique of applying compression through a plate at a fracture site.









     

    A 5-year-old girl presents with an 8-week history of pain and swelling in the right knee, right shoulder, and left elbow after



    Cutting cones Wolff's Law The Hueter-Volkmann Law Piezoelectric effect Compression plating

    Corrent answer: 3

    Limb buds develop at 4 weeks and are first able to be visualized by transvaginal ultrasound at 8 weeks.

    In a developing fetus, limb buds form at 26 days. The development of the limb is guided by a complex interaction of gene transcription factors and regulatory loops. The most important genes in limb development are sonic hedgehog (SHH), HOX genes, and WNT genes. Ultrasound evaluation is increasingly

    being utilized to diagnose and guide treatment for developmental anomalies of a fetus. The limb buds of the fetus can be first seen at 8 weeks of gestation.

     

    Krakow et al. reported on the prenatal diagnosis of fetal developmental dysplasias. They found that differentiating these disorders in the prenatal period can be challenging because they are rare and because many of the ultrasound findings are not necessarily pathognomic for a specific disorder.

     

    Oetgen et al. authored a review on prenatal diagnosis of musculoskeletal conditions. They note that ultrasonography is a safe and cost-effective tool used to prenatally detect common musculoskeletal conditions such as clubfoot, skeletal dysplasias, limb-length discrepancies, spinal abnormalities, and hand and other upper extremity deformities.

    Illustration A is a pictorial representation of limb bud formation Incorrect Answers:

    Answers 1,2,4,5: Each of these options incorrectly pairs the timing of limb bud formation with the ability to identify limb buds via ultrasonography.







     

    The hardware shown in Figure A relies on which of the following principles to achieve its function?


    Limb buds develop at 6 weeks and are first able to be visualized by a transvaginal ultrasound at 12 weeks Limb buds develop at 4 weeks and are first able to be visualized by a transvaginal ultrasound at 12 weeks Limb buds develop at 4 weeks and are first able to be visualized by a transvaginal ultrasound at 8 weeks Limb buds develop at 4 weeks and are first able to be visualized by a transabdominal ultrasound at 8 weeks Limb buds develop at 6 weeks and are first able to be visualized by a transvaginal ultrasound at 8 weeks
     

    Corrent answer: 5

    Both Hemophilia A and B are inherited by X-linked recessive patterns. Hemophilia A is caused by factor VIII deficiency, whereas hemophilia B is caused by factor IX deficiency.

     

    Factor VIII deficiency, also known as Hemophilia A, most commonly affects males due to the X-linked recessive inheritance pattern and occurs with a frequency of 1:5000 males. Factor IX deficiency, also known as hemophilia B, also only affects males due to X-linked recessive inheritance, with a frequency of 1:30000 males. Both disorders commonly present with recurrent spontaneous hemarthroses that affect large joints, typically the knee, leading to chronic synovitis and eventually joint destruction. Initial treatment involves factor replacement to within 60% normal, joint aspiration, and immobilization until the physical exam is normal. Treatment for chronic synovitis involves radiosynovectomy, open synovectomy, or arthroscopic synovectomy. End- stage joint destruction requires reconstructive surgery with aggressive factor replacement pre- and postoperatively.

     

    Luck et al. performed a review on hemophilic arthropathy and recommended treatment options for hemophilic arthropathy. The authors recommend that infants get

    primary prophylaxis with factor replacement prior to developing a "target" joint. In patients that experience a hemarthrosis, factor replacement with joint aspiration and immobilization until a normal physical exam are paramount for reducing chronic synovitis. Synovectomy, either arthroscopic or open, is recommended for chronic synovitis to reduce the progression of arthropathy. Then arthroplasty is reserved for end-stage joint destruction characteristic of recurrent synovitis.

     

    Zingg et al. performed a retrospective review of 43 consecutive TKA in patients with hemophilic arthropathy. At 9.5 years follow-up there were two hematogenous infections, three revisions, 94% good-to-excellent patient- reported outcomes, and significantly increased ROM compared to preoperative examination. The authors concluded that TKA remains a successful option for end-stage arthropathy for hemophiliacs, but outcomes do not reach the level of non-hemophiliacs.

     

    Journeycake et al. performed a retrospective review on 28 arthroscopic synovectomies performed on pediatric hemophiliac patients with chronic synovitis. At 5-years follow-up 76% of affected joints had stable or improved levels of function. The authors concluded that arthroscopic synovectomy provides a reliable means for limiting current hemorrhage in the affected joint and improving ROM.

     

    Illustration A depicts a pedigree with an X-linked recessive inheritance pattern. Notice how only males are affected, but women can be carriers. Illustration B depicts the process by which recurrent hemarthroses leading to chronic synovitis and then arthropathy.

     

    Incorrect Answers:

    Answer 1: Hemophilia A is inherited in an X-linked recessive pattern but of the answer choices it is not the only one.

    Answer 2: Hemophilia B is inherited in an X-linked recessive pattern but of the answer choices it is not the only one.

    Answer 3: Hemophilia C (factor XI deficiency), the fourth most common bleeding disorder, is inherited in an autosomal recessive pattern. Unlike hemophilia A and B, it does not cause spontaneous hemarthroses, but rather prolonged and excessive bleeding associated surgical procedures.

    Answer 4: Von Willibrand disease is the most common bleeding disorder and is due to the

    deficiency of von Willibrand factor; which assists in platelet adhesion. It is inherited in autosomal recessive pattern with the gene locus found on chromosome 12.






     

    In terms of fetal limb bud development, which of the following is true?


    Factor VIII deficiency Factor IX deficiency Factor XI deficiency Von Willibrand factor deficiency Both 1 & 2
     

    Corrent answer: 3

     

    Fluoroquinolones such as levofloxacin act by block DNA replication by inhibiting DNA gyrase.

     

    Fluoroquinolone antibiotics are bactericidal, and their mechanism of action works through the inhibition of DNA gyrase. Side effects of fluoroquinolones include inhibition

    of early fracture healing through toxic effects on chondrocytes and increased rates of tendinitis, with a special predilection for the Achilles tendon.

     

    Levine et al. published a review on fluoroquinolones. They report that fluoroquinolones act by inhibiting DNA topoisomerases such as DNA gyrase (topoisomerase II). Due to increasing antibiotic resistance, their use is limited to specific clinical scenarios.

    Additionally, their use in children is restricted due to a potential for growth disturbance and cartilage damage.

     

    Perry et al. performed an experimental study on the inhibition of fracture healing by levofloxacin and trovafloxacin in rats. They found that experimental fractures systemically exposed to levofloxacin or trovafloxacin have diminished healing during the early stages of fracture repair. They, therefore, concluded that the administration of quinolones during early fracture repair may compromise fracture healing in humans.

    Illustration A is an image illustrating the targets of the various antibiotic classes. Incorrect Answers:

    Answers 1, 4, and 5: The inhibition of cell wall synthesis is the mechanism of action of the cephalosporins, penicillin, and vancomycin.

    Answer 2: Aminoglycosides bind the 30S subunit of ribosomes, inhibiting protein synthesis from mRNA.





     

    Which of the following bleeding disorders is caused by an X- linked recessive mutation?


    Penicillins, inhibits bacterial protein synthesis Aminoglycosides, inhibits bacterial cell wall synthesis Fluoroquinolones, inhibits DNA gyrase Cephalosporins, inhibits bacterial protein synthesis Vancomycin, inhibits bacterial protein synthesis
     

    Corrent answer: 4

    An isotonic muscle contraction is a muscle contraction with constant tension such as the upwards and downwards motions of a biceps curl.

     

    The word “isotonic” is derived from two Greek words: “iso”, meaning “same”, and “tonikos” meaning “tension”. An isotonic muscle contraction is one during which the muscle maintains the same tension as it shortens. There are two types of isotonic contractions: concentric and eccentric. In a concentric isotonic contraction, the muscle shortens while contracting. In an eccentric isotonic contraction, the muscle lengthens during contraction.

     

    Ashe et al. review exercise programs used in physical therapy. They report that muscle strengthening can be classified into isotonic, isometric, and isokinetic contractions.

    Isotonic exercises involve the development of muscular force through range of motion or movement. Isokinetic exercises involve the force generation at a constant speed.

    Isometric exercises involve the development of force without movement, as in tensing and holding a muscle

    at a certain part of the range.

     

    Illustration A is an image which illustrates the difference between isotonic and isometric contractions.

     

    Incorrect Answers:

    Answers 1 & 2: The upwards and downwards motion of a biceps curl are both examples of isotonic muscle contractions

    Answers 3 & 5: Pushing against a wall is an example of an isometric muscle contraction.







     

    Which of the following correctly describes a class of antibiotics and its mechanism of action?


    Upwards motion of biceps curl Downwards motion of biceps curl Pushing against a wall Answers 1 and 2 Answers 3 and 2
     

    Corrent answer: 1

     

    The third step in applying EBM is to appraise the evidence.

     

    Evidence-based medicine (EBM) refers to an explicit process of using and evaluating information to make medical decisions. When applying EBM in practice, there are 5 steps that should be followed: 1) formulate an answerable question, 2) gather evidence, 3) appraise the evidence, 4)

    implement the evidence, and 5) evaluate the process to determine the efficacy of the proposed treatment.

     

    Bernstein published a review on EBM. He advocates for the use of a five-step process for sound decision making: formulate answerable questions, gather evidence, appraise the evidence, implement the valid evidence, and evaluate the process.

    Spindler et al. published a review on reading and reviewing the orthopaedic literature. They focus on the third step of EBM: appraising the evidence. They report that systematic review assists the orthopaedic surgeon in interpreting study results and in understanding the relative validity of these results in the hierarchy of evidence.

    Illustration A is a table listing the 5 steps of EBM.

    Incorrect Answers:

    Answer 2: Formulating an answerable question is the first step of EBM. Answer 3: Implementing evidence is the fourth step of EBM. Answer

    4: Gathering evidence is the second step of EBM.

    Answer 5: Evaluating the process is the fifth and final step of EBM.












     

    Which of the following activities describes an isotonic muscle contraction?


    Appraise the evidence Formulate an answerable question Implement the evidence Gather the evidence Evaluate the process
     

    Corrent answer: 2

     

    On average, physicians interrupt patients within 23 seconds of their interview.

     

    The patient-physician interaction often begins with an initial "survey of problems" through an open-ended question designed to give the patient the freedom to speak and explain

    their reason for seeking care. This is followed by a set of focused or closed-ended questions designed to elicit additional details and clarification. Unfortunately, physicians are often quick to interrupt or redirect patients prior to the completion of their reason for seeking care. This practice may lead to missed information, poor communication, and poor

    patient satisfaction. Time constraints on physicians may contribute to this behavior. Marvel et al. looked at 264 patient-physician interviews by board-certified family practice physicians. They found patients' initial statement of concern were only complete 28% of the time with an average physician redirection

    time of 23.1 seconds. They found patients only needed an additional 6 seconds to complete their statement of concern compared to those who were

    redirected by the physician. They conclude that obtaining a complete patient agenda takes little time and can improve interview efficacy and increase data collection.

     

    Incorrect Answers:

    Answer 1: On average, physicians give the patients more than 7 seconds to provide their initial statement of concern.

    Answer 3, 4, & 5: These times are longer than the average time that physicians interrupt patients.





     

    When applying evidence-based medicine (EBM) in practice, there are 5 steps that should be followed. Which of the following describes the third step?


    7 seconds 23 seconds 46 seconds 2 minutes 1 minute
     

    Corrent answer: 5

     

    For upper extremity surgery, the majority of narcotic pills prescribed by hand surgeons are not consumed by patients.

     

    Patients in the United States are treated very aggressively for pain control. This is due, in part, to The Joint Commission, which has controversially identified pain as the "5th vital sign." An opioid epidemic has ensued which has been linked to a decreased the life expectancy in the United States for three consecutive years beginning in 2014. As a result, unused prescription pain medication following upper extremity surgery represents a liability for patients and surgeons. Following simple soft tissue surgeries (trigger finger, carpal tunnel, mass removal) patients typically only require pain

    medication for 2-3 days, and there is no difference in pain control between narcotic or anti-inflammatory medication.

     

    Stanek et al. implemented a standardized postoperative opioid prescription protocol for a group of academic hand surgeons. They found that the protocol decreased the opioid prescription size by 15%, prescription variability, and decreased refills. The authors recommend the development of specific prescription guidelines.

     

    Rodgers et al. interviewed 250 patients after upper extremity surgery about opioid consumption. They reported that patients most frequently received 30 narcotic pills, which provided relief in 92% of cases. The authors found that patients undergoing bone procedures used on average 14 pills, most patients took medication for less than two days post-operatively, and the number of pills consumed on average was 10, with 19 pills unused. As a result, the authors advocated for more limited narcotic prescriptions post-operatively.

    Weinheimer et al. randomized patients undergoing hand surgery to receive either Norco or acetaminophen/ibuprofen. They found no difference in time until patients were pain- free, average VAS scores, or the absolute number of those patients who were pain-free. They did find that the narcotic group experienced more adverse side effects (23% vs 3%), ultimately recommending for limiting narcotics post-operatively.

     

    Incorrect answers:

    Answer 1: A standardized protocol has been shown to reduce variability and the number of pills prescribed.

    Answer 2: There is no significant difference in pain control between narcotics and anti- inflammatory medications for simple hand cases.

    Answer 3: Following simple hand surgery, patients who are prescribed narcotics have been shown to experience more adverse side effects as compared to those who were prescribed anti-inflammatory medications. Answer 4: The duration of pain is no different between narcotic and anti- inflammatory cohorts in hand surgery.





     

    During a new patient office visit, a physician asks an initial open- ended question to the patient. On average, how much time elapses before the physician redirects the patient's initial statement of concern?


    A prescription protocol does not affect prescription patterns Narcotic medication will offer superior pain relief as compared to anti- inflammatory medication Anti-inflammatory medication leads to more side effects than narcotic medication The duration of post operative pain is longer with anti-inflammatory medication than with narcotic medication Over half of opioid pills prescribed by hand surgeons are not used

    Corrent answer: 2

     

    The patient’s laboratory workup is likely to reveal hypovitaminosis D. Metabolic and endocrine abnormalities should be considered in patients who exhibit poor fracture healing, especially in those who lack history and exam findings suggestive of infection.

     

    In addition to ruling out infection, a nonunion workup should include tests to identify metabolic and endocrine abnormalities. 25-hydroxyvitamin D3, synthesized from cholecalciferol in the liver, is the laboratory study of choice to determine vitamin D deficiency. Low vitamin D is a common, and easily treated, form of malnourishment in orthopedic trauma patients. Other important factors that can negatively impact fracture healing include protein malnourishment, diabetes mellitus, nicotine use, and HIV.

    Warner et al. showed perioperative vitamin D deficiency correlated with

    inferior clinical outcomes in patients who underwent operative fixation of ankle fractures. Of the 98 patients studied, 36 (37%) were found to be deficient in vitamin D (<20 ng/ml) and 31 (32%) were found to have a vitamin D insufficiency (> 20 ng/ml, <30 ng/ml). They concluded insufficient vitamin D may result in worse outcomes in orthopedic trauma patients recovering from fracture fixation.

     

    Brinker et al. reviewed metabolic and endocrine abnormalities in 37 patients with nonunions. Inclusion criteria were: 1) an unexplained nonunion that occurred despite adequate reduction and stabilization; 2) history of multiple low-energy fractures with at

    least one nonunion; or 3) a nonunion of a nondisplaced pubic rami or sacral ala fracture. Of the 37 patients who met screening criteria, 31 (84%) had metabolic or endocrine abnormalities. Vitamin D deficiency, discovered in 25 of 37 patients (68%), was the most common newly diagnosed abnormality. The authors conclude all patients with nonunion who meet their screening criteria should be referred to an endocrinologist.

     

    Bishop et al. reviewed the assessment of compromised fracture healing and advocate for a metabolic and endocrine workup in the presence of nonunion. If an endocrinology consultation is unavailable, the initial laboratory screening should include 25- hydroxyvitamin D, calcium, thyroid-stimulation hormone, phosphorus, and alkaline phosphatase levels. They also emphasize that the presence of normal inflammatory markers does not exclude the possibility of infection, which should remain in consideration until fracture union and resolution of symptoms.

     

    Incorrect Answers

    Answer 1: An arrest in the fracture repair process should prompt further workup. If workup is unrevealing, continued observation with or without fracture brace immobilization and/or a bone stimulator can be considered. Answer 3: Weight-bearing should be continued to stimulate additional fracture healing.

    Answer 4: Initiation of suppressive antibiotics may be useful in the treatment of osteomyelitis in patients who are immunocompromised or those who are unable to undergo surgery.

    Answer 5: Hardware removal, debridement, antibiotic bead placement, and IV antibiotics should be considered when an infection is thought to be the cause of a nonunion.

    However, the rate of infection is lower than hypovitaminosis D and both can occur simultaneously.

     

    A hand surgeon plans on performing a carpal tunnel release on a healthy 45- year-old female. Which of the following is true regarding pain management for this case in the post-operative setting?


    Continued observation without any intervention Continued observation and vitamin D supplementation Vitamin D supplementation and make non-weight-bearing Continued observation with suppressive antibiotics Hardware removal, debridement, antibiotic bead placement, and initiation of organism specific IV antibiotics

    Corrent answer: 4

     

    The ideal scenario to use the ANOVA test is when comparing parametric continuous data (i.e. BMI) for three or more groups.

     

    In statistical analyses, data can be described as discrete (categorical, ordinal) or continuous. Discrete data are observations that can be expressed as categories such as gender, race, or disease status. Continuous data, such as age, are observations for which the difference between the numbers have meaning on a numerical scale. The ANOVA test is used to compare differences in mean values (i.e. continuous data) when there are more than two independent sample groups. When discrete data is compared in the setting of two or more independent sample groups, the chi-squared (parametric) or Fischer's exact test (non-parametric) may be utilized.

     

    Kuhn et al. reviewed statistical tests when discrete data are analyzed. They reported that data may be either discrete (i.e. categorical) or continuous (i.e. age, BMI, height). They presented examples of tests used for discrete data including the chi-square test and Fischer's exact test. They emphasized the importance of scrutinizing the data presented prior to selecting a statistical test.

     

    Greenfield et al. reviewed statistical tests when continuous data are analyzed. They reported that statistical tests for continuous data must be used if the outcome of interest is a comparison of sample means for data that are continuous (i.e. the height of populations). They discuss one-sample t-tests, independent two-sample t-tests, paired t- tests, and ANOVA.

     

    Illustration A demonstrates an algorithm that is helpful in selecting the correct statistical test.

     

    Incorrect Answers:

    Answers 1: The proportion of patients in each group is discrete data and requires the chi- squared test if the data is parametric or non-parametric. Answer 2: The proportion of patients in each cohort is discrete data and requires Fischer's exact test if the data is parametric or non-parametric with a small sample size.

    Answer 3: When comparing three or more independent groups and the data is non- parametric, the Kruskal-Wallis test should be utilized.

    Answer 5: When comparing the average age of patients in two cohorts when the data is non-parametric, the Mann-Whitney U-test should be utilized.







     

    A 40-year-old Hispanic male presents with persistent pain seven months after open reduction internal fixation of a closed distal tibial fracture. His postoperative course was unremarkable and weight- bearing was resumed at six weeks. Exam reveals a well-healed incision with tenderness at the fracture site. There is no swelling or erythema. Radiographs demonstrate intact hardware and an oligotrophic nonunion. Laboratory workup is most likely to support which of the following interventions:

    To compare the proportion of patients undergoing hip pinning, hemiarthroplasty, and total hip arthroplasty when the data is parametric To compare the proportion of patients undergoing hip pinning, hemiarthroplasty, and total hip arthroplasty when the data is non-parametric To compare the average age of patients undergoing hip pinning, hemiarthroplasty, and total hip arthroplasty when the data is non-parametric To compare the average BMI of patients undergoing hip pinning, hemiarthroplasty, and total hip arthroplasty when the data is parametric To compare the average age of patients undergoing hip hemiarthroplasty versus total hip arthroplasty when the data is non-parametric
     

    Corrent answer: 2

     

    General anesthesia carries an increased risk of thromboembolism compared to spinal anesthesia. The remaining statements are false.

     

    There have been multiple factors that demonstrate an increased risk of venous thromboembolism (VTE). Some of these risk factors include a previous VTE, obesity (BMI

    > 30), surgery type (i.e. total joint arthroplasty),

    hypercoagulable states (i.e. cancer, inheritable traits), myocardial infarction (MI), congestive heart failure, family history of VTE, hormone replacement therapy, elevated hormone conditions, varicose veins, and general anesthesia (increased risk compared to epidural/spinal anesthesia). Current AAOS guidelines recommend mechanical prophylaxis in all total hip and knee arthroplasty patients and chemoprophylaxis is recommended, but no optimal regimen is recommended. Chemical prophylaxis should be individualized for each patient and their risk factors.

    Geerts et al. put forth their recommendations on the prevention of VTE from the American College of Chest Physicians in 2008. Some of the important points include aspirin not being recommended as a monotherapy (this recommendation was changed in 2012 and is now accepted as monotherapy), recommendation for mechanical prophylaxis, and recommendation for routine chemoprophylaxis for elective hip and knee arthroplasty for a minimum of 10 days.

     

    Caprini et al. retrospectively reviewed 939 patients with either a DVT, PE, or PE and DVT and their treatment. They found that there was lower than anticipated use of low molecular weight heparin, insufficient bridging of patients to warfarin, and insufficient continuation of anticoagulation following hospitalization. They concluded that hospitals need to re-evaluate adherence to VTE treatment guidelines and develop strategies to optimize therapy.

     

    Incorrect Answers:

    Answer 1: MTHFR mutation carries one of the highest risks of thromboembolism.

    Answer 3: The use of erythropoietin increases the risk of thromboembolism. Answer 4: Oral tranexamic acid use has not been shown to increased the risk of thromboembolism.

    Answer 5: Hormone replacement therapy increases the risk of thromboembolism.







     

    An orthopedic surgery intern is preparing to perform statistical analyses for his research project. He presents to the department statistician inquiring about the Analysis of Variance (ANOVA) test. Which of the following below is the ideal scenario to use the ANOVA test?


    MTHFR mutation has no impact on thromboembolism risk Intraoperative conversion from spinal to general anesthesia increases thromboembolism risk Post-operative anemia treated with erythropoietin decreases thromboembolism risk Oral tranexamic acid use increases thromboembolism risk Post-menopausal estrogen use is protective from thromboembolism

    Corrent answer: 1

     

    While almost all patients undergoing major orthopaedic procedures receive VTE prophylaxis, this is often not within the ACCP post-operative VTE prophylaxis guidelines.

     

    VTE events typically occur following hospital discharge, within the first 2 to 6 weeks following surgery. Risk is increased with major orthopaedic surgery due to greater soft tissue disruption, venous stasis from limb manipulation, and post-operative immobility. VTE following major orthopaedic hip and knee surgery represents not only a significant cause of postoperative morbidity and mortality but contributes a substantial economic burden. Prophylaxis is the single-most-important factor mitigating the risk of VTE. Therefore adherence to the AAOS and ACCP guidelines is recommended.

     

    Friedman et al. evaluated compliance of physicians with American College of Chest Physicians (ACCP) post-operative VTE prophylaxis guidelines following TKA and THA. The authors found moderate compliance overall, with 47% of THA and 61% of TKA patients receiving appropriate prophylaxis in terms of type, duration, start time, and dose. Compliance with Warfarin use was the lowest, while that with low-molecular-weight heparin was significantly higher. They concluded that while almost all patients received prophylaxis, this was often not within the guidelines.

     

    Oster et al. investigated the economic consequences of VTE following major orthopaedic hip or knee surgery. The authors found that 2.2% of patients developed clinically significant VTE, 62% after hospital discharge and that patients who developed in-hospital VTE had a significantly longer length of hospital stay and associated costs than those that did out-of-hospital and were later readmitted. They concluded that the economic impact of VTE was

    substantial regardless of setting.

     

    Incorrect answers:

    Answer 2: The length of hospitalization and associated medical costs are significantly increased by VTE whether occurring in-hospital or following discharge

    Answer 3: Most acute perioperative pulmonary events are due to fat embolism or cardiac events.

    Answer 4: Most VTE events have been found to occur following hospital discharge, between 2 and 6 weeks following total joint arthroplasty.

    Answer 5: LMWH prophylaxis was fully compliant with ACCP guidelines more often than Warfarin prophylaxis for both THA (63% versus 33%) as well as TKA (72% versus 48%).





     

    Which of the following statements is true as it relates to the risk of thromboembolism?


    Almost all patients receive VTE prophylaxis, though often not in compliance with the American College of Chest Physicians (ACCP) guidelines Length of hospitalization and associated medical costs are significantly increased by in-hospital VTE but not post-discharge VTE Most acute perioperative pulmonary events result from dislodged chronic deep venous thrombosis resulting in pulmonary embolism Most symptomatic VTE occur within two weeks after total joint arthroplasty Warfarin prophylaxis is more often fully compliant with ACCP guidelines than low- molecular-weight heparin (LMWH) prophylaxis

    Corrent answer: 4

     

    Those utilizing opioids prior to elective hip and knee arthroplasty are at elevated risk of complications. Weaning opioids preoperatively has been shown to improve postoperative outcomes.

     

    Nonoperative management of osteoarthritis (OA) is focused on reducing pain and limiting functional impairment with medications, physical therapy, activity modification, weight

    loss, and intra-articular corticosteroid injections.

    Pharmacologic management of OA includes NSAIDs and tramadol (per the AAOS CPGs). Opioids have been increasingly used for OA despite the lack of evidence behind their usage. Chronic opioid usage may improve OA-related pain but it has been associated with numerous adverse effects and worse musculoskeletal treatment outcomes.

     

    Gaffney et al. in their review of perioperative pain management for hip and knee arthroplasty, describe the role of opioids, cryotherapy, acetaminophen, NSAIDs, tramadol, anticonvulsants, spinal analgesia, epidural analgesia, peripheral nerve blocks, and periarticular injections. They recommend IV dexamethasone on POD1, scheduled Tylenol, scheduled NSAIDs (Celebrex vs naproxen vs ketorolac), and PRN tramadol, oxycodone, or hydromorphone for breakthrough pain.

     

    Nguyen et al. performed a retrospective matched cohort comparing patients undergoing hip or knee arthroplasty who were either opioid-naive, chronic opioid users, or chronic opioid users weaned of opioids preoperatively. They found that chronic opioid users who were able to reduce their preoperative opioid use by half prior to arthroplasty had outcomes (SF12 physical component and WOMAC) superior to those who were unable to decrease preoperative opioid use.

     

    Sing et al. performed a retrospective review including 1263 patients undergoing primary THA or TKA, finding that patients who utilized opioids preoperatively had elevated postoperative pain, consumed a greater amount of morphine equivalents, walked fewer meters, had a longer postoperative length of stay, were more likely to be discharged to a care facility, and had 4-

    5x greater 90d complications. They conclude that opioid users are a high-risk group.

     

    Incorrect answers:

    Answer 1: Weaning opioids prior to joint arthroplasty leads to improved postoperative function, pain, and recovery.

    Answer 2: Weaning opioids does not have a negative effect on surgical outcomes.

    Answer 3: Weaning opioids improves outcomes compared to those who do not wean, and also has a similar improvement in outcomes to the opioid-naïve. Answer 5: Weaning opioids does improve outcomes compared to not weaning, but weaning opioids does not achieve superior outcomes compared to the opioid-naïve.




     

    Which of the following is true regarding venous thromboembolism (VTE) following major orthopaedic hip or knee surgery?


    No effect Worse outcome Improved outcome compared to if he had not weaned from opioids, but minimal improvement compared to if he were opioid naive Improved outcome compared to if he had not weaned from opioids, and similar improvement in function compared to if he were opioid naive Improved outcome compared to if he had not weaned from opioids, and better outcomes compared to if he were opioid naive
     

    Corrent answer: 3

     

    This patient has developed CRPS following fixation of a distal radius fracture. All of the above are characteristics of CRPS except for decreased perfusion to the fingertips.

     

    CRPS is divided into two general categories: Type I, occurring in the absence of a specific nerve injury, and Type II, resulting from presence of a specific nerve injury. The incidence is 6-26 cases per 100,000 person-years, mostly affecting females (4:1), and smokers. The International Association of the Study of Pain (IASP) has developed the Budapest Criteria for the diagnosis of CRPS. These include sensory (reports of hyperesthesia and/or allodynia), vasomotor (reports of temperature asymmetry and/or skin color changes and/or skin color asymmetry), sudomotor/edema (reports of edema and/or sweating changes and/or sweating asymmetry), and motor (reports of decreased ROM, weakness, or trophic changes to hair or nails) changes. Vitamin C following distal

    radius fracture fixation has been suggested as preventative, though still somewhat controversial. Following development of CRPS, treatment includes psychotherapy, occupational therapy, sympathetic blockade, and antidepressants.

     

    Birklein and Schlereth comprehensively review CRPS. The authors describe how after a trauma there is an abundance of inflammatory mediators which stimulate the peripheral nerves. In addition, the proinflammatory cytokine network stimulates bone cell and fibroblast proliferation and potentially endothelial dysfunction. They note that these molecular changes lead to autonomic disturbances and an overwhelming pain response.

    Koh et al. also present a review of CRPS. The authors stress that CRPS is a clinical diagnosis and one of exclusion. They discuss that CRPS is best treated within a multidisciplinary team including orthopaedic surgeons, pain management, therapy, and psychological services. Early diagnosis is furthermore critical. Finally, the authors advocated vitamin C administration on the day of fracture as prophylaxis against CRPS.

     

    Incorrect answers:

    Answer 1: Hyperesthesia and allodynia can be diagnostic signs of CRPS. Answer 2: Skin temperature and color asymmetry as compared to contralateral wrist can be diagnostic signs of CRPS.

    Answer 4: Edema and sweating asymmetry can be diagnostic signs of CRPS. Answer 5: Decreased range of motion and weakness can be diagnostic signs of CRPS.







     

    A 65-year old male with worsening right hip osteoarthritis has failed nonsurgical management and would like to proceed with total hip arthroplasty. He has consulted with a pain management specialist and is treating his pain with opioids. If he is able to successfully decrease the amount of opioids he takes preoperatively by 50%, what effect would that have on his postoperative functional outcome?


    Hyperesthesia and allodynia Skin temperature and color asymmetry as compared to contralateral wrist Decreased perfusion to fingertips Edema and sweating asymmetry Decreased range of motion and weakness
     

    Corrent answer: 5

     

    Etanercept is a biologic disease modifying anti-rheumatic drug (DMARD) which works by binding and inhibiting TNF-a, in effect suppressing the autoimmune response associated with rheumatoid arthritis (RA).

     

    There are a number of DMARDs commonly used in the medical management of RA. TNF- a is a frequent target, given its pivotal role as one of the major cytokines driving the progression of RA. Etanercept is one example of a TNF-a inhibitor that is often used to treat RA, juvenile RA, psoriatic arthritis, and ankylosing spondylitis. Infliximab, adalimimab, golimumab are other

    commonly used TNF-a inhibitors. Before initiating these medications, patients and physicians should be aware of the possibility of reactivation of latent tuberculosis as well

    as increased rates of infection and lymphomas with long- term use.

     

    Saleh et al. reviewed the perioperative management of RA patients. They note that patients that are maintained on etanercept perioperatively have a significantly increased rate of perioperative infection. The authors discuss recommendations that etanercept be held at least one half-life prior to surgery and in some instances up to 4-5 half-lives before surgery. They recommend restarting the medication at 2 weeks post-operatively so long as the surgical sites are healing uneventfully.

     

    Nikiphorou et al. evaluated the impact of biologic agents on the surgical treatment of RA. The authors discussed that although rates of major joint replacements (THA/TKA) for osteoarthritis are increasing, the rates of THA/TKA for RA has been essentially unchanged over >10 years. They concluded that effective medical management of RA has led to fewer orthopedic surgeries being performed in the RA population.

     

    Incorrect Answers:

    Answer 1: Methotrexate is a folic acid antagonist that works through the inhibition of dihydrofolate reductase.

    Answer 2: Rituximab is a monoclonal antibody to the CD20 antigen. Answer 3: Anakinra is a recombinant IL1 receptor antagonist.

    Answer 4: Abatacept is selective co-stimulation modulator that binds to CD80 and CD86, in effect inhibiting T cells.

     

    A 60-year-old female underwent open reduction and internal fixation of a distal radius fracture 3 weeks ago. She returns to your clinic and appears anxious. She complains of pain and difficulty sleeping. When you remove her splint her entire hand and wrist are sensitive. You suspect that she has developed complex regional pain syndrome (CRPS). All of the following are common signs or symptoms of CRPS EXCEPT:

    Methotrexate Rituximab Anakinra Abatacept Etanercept

    Corrent answer: 2

     

    All of the statements listed above are true EXCEPT for answer 2 - BMP-2 is not FDA indicated for single-level posterolateral lumbar fusions.

     

    Bone morphogenetic proteins are a member of the TGF-beta superfamily. It is an osteoinductive material that induces mesenchymal stems cells to differentiate into bone- forming osteoblasts. There has been an increasing amount of literature published around its use in long bone procedures, spinal procedures, and nonunions. Currently, the FDA indications for rhBMP-2 are acute open tibial shaft fractures treated within 14 days and single level ALIFs with a lumbar tapered fusion device.

     

    Hsu et al. authored a systematic review including 6 articles on the cost- effectiveness of BMP-2 compared to iliac crest bone graft (ICBG) in lumbar and cervical arthrodesis procedures. They conclude that in lumbar arthrodesis procedures BMP-2 is only cost- effective when taking into account societal costs such as productivity and lost wages.

     

    Carreon et al. performed a cost-utility analysis on an RCT that they performed comparing BMP-2 to ICBG in posterolateral lumbar fusions. There are more complications, increased need for additional treatment and revision surgery in patients over 60 years old receiving ICBG compared with rhBMP-2/ACS, which account for an increased cost utility for the ICBG group.

     

    Glassman et al., in the paper that the aforementioned study worked off of, performed an RCT of rhBMP-2/ACS (Infuse bone graft) versus iliac crest bone

    graft (ICBG) for lumbar spine fusion in patients over 60 years of age. They conclude that BMP-2 is a viable ICBG replacement in older patients in terms of safety, clinical efficacy, and cost-effectiveness.

     

    Cheng et al. looked at the osteogenic activity of fourteen different BMPs on mesenchymal progenitor cells. They found BMP-2, 6, and 9 induced high levels of alkaline phosphatase activity in pluripotent stem cells. They conclude BMP-

    2, 6, and 9 may play important roles in inducing osteoblast differentiation of mesenchymal stem cells.

    Illustration A (Cheng et al.) is a figure demonstrating the distinct osteogenic activity of human BMPs. BMP-2, 6, and 9 are the most potent agents to induce osteoblast lineage differentiation of mesenchymal progenitor cells while most BMPs can promote the terminal differentiation of committed osteoblast precursors.

     

    Incorrect answers:

    Answers 1: BMP-2 is currently FDA indicated for acute open tibial shaft fracture. Answers 3: As noted in the literature above, rhBMP-2 has been shown to be cost- effective when utilized for posterolateral fusions in patients over 60 years old when taking into account the societal costs associated with increased need for revision surgery when utilizing ICBG.

    Answers 4: BMP-2 is currently FDA indicated It is FDA indicated for single level ALIFs

    Answer 5: As the studies note above, in patients older than 60 years old, the use of ICBG was associated with an increased need for revision surgery









     

    Which of the following medications specifically target tumor necrosis factor alpha (TNF-a)?


    It is FDA indicated for acute open tibial shaft fracture It is FDA indicated for single level posterolateral lumbar fusions It is cost-effective for posterolateral lumbar fusions when compared to iliac crest bone graft It is FDA indicated for single level ALIFs It has a decreased risk of revision surgery when compared to iliac crest bone grafting in posterolateral lumbar fusions
     

    Corrent answer: 2

     

    Fretting corrosion results from the relative micromotion between two affixed materials placed under a load and is characterized by the formation of pits,

    grooves, and oxide debris. This may be seen at modular junctions.

     

    The process of fretting corrosion involves the physical disruption of the passivated layer at the junction of two materials due to friction caused by micromotion under pressure.

    The increased surface roughness and release of metallic oxide debris may then, in turn, lead to other types of corrosion such as crevice corrosion. Fretting corrosion has been described at the head-neck junction in total hip arthroplasty, and the risk is increased with an increasing number of component interfaces.

     

    Brown et al. describe fretting corrosion within the context of modular hip tapers. The authors note that while modularity increases versatility, this comes at the cost of interfacial corrosion which may result in both device failure as well as the release of metal ions with local soft tissue reactions. They conclude that longer neck extension was associated with increased fretting corrosion

    and that this can potentially be mitigated by increasing the stability of the interface.

     

    Goldberg et al. performed in vitro corrosion testing of modular hip tapers. The authors found that once fretting corrosion created an environment suitable for crevice corrosion, corrosion continued regardless of continued mechanical loading. They concluded that mechanical loading had a significant impact on initiating the corrosion process.

     

    Incorrect Answers:

    Answer 1: Crevice corrosion is a localized electrochemical process that results from trapping and stagnation of a corrodent between either two metal surfaces or metal and non-metal surfaces. The low oxygen content within the crevice creates a relative electronegative environment compared to the environment outside of the crevice, which acts as a cathode due to higher oxygen content and pH.

    Answer 3: Galvanic corrosion is an electrochemical reaction resulting from two metals of dissimilar electrode potentials in electrical contact with one another through a conductive electrolyte media. One metal is a relative anode and the other a cathode. This is generally not an issue for metals with <0.15V difference in anodic index even in harsh environments.

    Answer 4: Pitting corrosion is an autocatalytic process that results from depassivation of small areas of metal with the formation of small holes or “pits”. This is a localized galvanic corrosion process in which the small area acts as a relative anode and the metal surface as a cathode.

    Answer 5: Stress corrosion or stress corrosion cracking is the formation of  fine microscopic cracks resulting from high tensile stress combined with a corrosive environment and elevated temperatures. This may also result from rapid

    temperature cycling causing rapid expansion/contraction of the metal. This may be a consideration during the manufacturing process of implants but is not seen in vivo.

    When considering using recombinant human BMP-2 in orthopaedic surgery, all of the following are true EXCEPT:


    Crevice Fretting Galvanic Pitting Stress
     

    Corrent answer: 2

     

    The null hypothesis in this randomized controlled trial is that there is no difference in cement penetration during TKA with or without tourniquet use. As there was significant crossover (tourniquet use in the "no tourniquet" cohort), accepting the null hypothesis when it is false would result in beta (type 2) error.

     

    In hypothesis testing, the assertion that the observed findings did not occur by chance alone but rather occurred because of a true association between variables is confirmed or rejected. By convention, the null hypothesis suggests that there is no significant association between variables while the alternative hypothesis suggests that there is a significant association. Alpha (type 1) error occurs when the null hypothesis is rejected

    when it is, in fact, true (false positive effect). Beta (type 2) error occurs when the null hypothesis is

    accepted when it is, in fact, false (false negative effect).

     

    Kocher et al. reviewed power analyses, statistical errors, and the concept of statistical power. They discuss that beta represents the chance of a type II error, while alpha represents the chance of a type I error, and that conventionally beta is set at 0.2 and alpha at 0.05. The authors recommended that when a study observes no difference, the power of the study, or (1 - beta), should be reported.

     

    Lochner et al. investigated the rates of beta error in randomized controlled trials in orthopedic trauma. They reported a 90% beta error rate in these trials, which exceeds accepted standards. The authors recommended that future authors perform pre-study power and sample-size calculations to

    reduce these rates.

     

    Illustration A shows a Bayesian analysis table demonstrating the relationship between alpha, beta, and the null hypothesis.

     

    Incorrect Answers:

    Answer 1: The alpha (type 1) error occurs when the null hypothesis is rejected when it is, in fact, true (false positive effect). In other words, a difference is found by chance when there is not one. In this scenario, the null hypothesis is accepted.

    Answer 3: (1 - alpha) corresponds to the probability of finding a true negative, i.e. the null hypothesis is correctly accepted.

    Answer 4: (1 - beta), or power, is the probability of finding a significant association if one truly exists.

    Answer 5: The kappa statistic is used to measure inter-observer and intra- observer reliability.









     

    Which of the following types of corrosion is defined by the formation of pits, grooves, and oxide debris due to the relative micromotion between two affixed materials placed under a load?


    alpha error beta error 1 - alpha 1 - beta kappa statistic

    Corrent answer: 1

     

    Tobramycin is an aminoglycoside that acts primarily by disrupting protein synthesis through irreversibly binding to 30S ribosomal subunit, leading to altered cell membrane permeability, disruption of the cell envelope, and eventual cell death.

     

    Exchange nailing with an antibiotic-impregnated intramedullary nail is often implemented in the treatment of chronic osteomyelitis with septic tibial nonunion as it provides both fracture stabilization and antibiotic elution. Vancomycin and tobramycin are often added to the polymethylmethacrylate (PMMA) cement for broad-spectrum coverage.

    Vancomycin disrupts cell-wall synthesis in a time-dependent manner by binding to the D- Ala-D-Ala terminal of the growing peptide. It is extremely effective in gram-positive bacteria, but ineffective against gram-negative bacilli due to its large size. Conversely, tobramycin is effective against gram-negative organisms, and works chiefly through the inhibition of bacterial protein synthesis by irreversibly binding to the 30S ribosomal subunit.

    Jaeblon et al. reviews the contemporary use of PMMA in orthopaedic surgery. The authors discuss the utility of PMMA as a delivery vehicle for antibiotics, eluting from both the surface and pores of the cement as well as the microcracks within it, while simultaneously eliminating dead space. They conclude that tobramycin is popular because it comes in powder form, which is easy to mix, and because of its broad spectrum activity, which includes antipseudomonal coverage. Moreover, it has been shown to potentiate the elution of other antibiotics, such as vancomycin.

     

    McNamara et al. reviews the mechanism of Vancomycin. The authors report how this antibiotic has increased in importance in the last decade due to the growing resistance of many gram-positive bacteria to β-lactam antibiotics. They discuss that vancomycin is a large, complex, tricyclic glycopeptide molecule that works primarily through disruption of the biosynthesis of peptidoglycan, the major structural polymer of the gram-positive

    bacterial cell walls, through binding to the D-alanyl-D-alanine terminal of cell wall precursor units. The authors conclude that unlike penicillins and cephalosporins, cross- resistance with vancomycin does not develop, because vancomycin acts against different stages of cell wall synthesis and different specific targets.

     

    Nana et al. discusses the high affinity of microorganisms to adhere to foreign materials commonly used in orthopedics, including cobalt-chromium, titanium, polyethylene, and PMMA cement through the formation of biofilms. The

    authors report that S. aureus and S. epidermidis are the most common biofilm-forming bacteria found in orthopedics, and, when combined with P. aeruginosa, they represent nearly 75% of biofilm infections. They conclude that while no current guidelines exist for treating these infections, recent studies have shown that biofilm growth can be fully inhibited when PMMA is mixed with both daptomycin and gentamicin.

     

    Figures A and B are the AP and lateral radiographs of an infected nonunion of a tibial shaft fracture treated initially with an intramedullary nail.

    Figure C is an axial CT image illustrating the tibial fracture nonunion.

     

    Incorrect Answers:

    Answer 2: Tobramycin binds to the 30s ribosomal subunit, however it does so irreversibly.

    Answer 3: Tobramycin binds irreversibly, but to the 30s ribosomal subunit. Answer 4: Tobramycin irreversibly binds to the 30s ribosomal subunit. Answer 5: This is the mechanism of action for Vancomycin.

     

    A randomized controlled trial is undertaken to investigate whether tourniquet use increases cement penetration during total knee arthroplasty. Approximately 40% of the patients that were initially randomized to the "no tourniquet" group had tourniquets placed intraoperatively due to difficulty with visualization. Intent- to- treat analysis was conducted and the results showed no difference in the rates of cement penetration. What statistical term best applies if these results are accepted at face value?




    Irreversibly binds to 30S ribosomal subunit, inhibiting bacterial protein synthesis Reversibly binds to 30S ribosomal subunit, inhibiting bacterial protein synthesis Irreversibly binds to 50S ribosomal subunit, inhibiting bacterial protein synthesis Reversibly binds to 50S ribosomal subunit, inhibiting bacterial protein synthesis Inhibits cell wall synthesis by binding to the D-alanyl-D-alanine precursor units
     

    Corrent answer: 1

     

    Sclerostin is a direct antagonist of the Wnt/β-catenin pathway and thus a key regulator of the formation of mineralized bone matrix and bone mass. Anti- sclerostin antibodies result in inactivation of sclerostin, thereby promoting the anabolic Wnt/β-catenin pathway and resulting in INCREASED bone density

     

    Sclerostin is a glycoprotein encoded by the SOST gene and produced primarily by osteocytes. It acts as a negative regulator of bone mass by directly antagonizing Wnt binding to the LRP5/6 receptor, thereby leading to β-catenin degradation and reduction of Wnt-target gene expression. This results in anti- anabolic properties, including inhibition of osteoblastic differentiation, bone formation, and loss of inhibition of osteoblast and osteocyte apoptosis. Genetic mutations resulting in loss of function or decreased expression of SOST have been linked to endosteal hyperostosis, increased bone mass, and increased bone density (as with Van Buchem disease and sclerosteosis). As a result, new immunotherapies targeting sclerostin (such as Romosozumab and

    Blosozumab) are being investigated for their utility toward treating osteoporosis and have shown promising results.

     

    Recker et al. presented a double-blinded phase 2 randomized controlled trial of blosozumab in the treatment of low bone mineral density in postmenopausal women. The authors found that administration of the monoclonal antibody resulted in significant dose- dependent increases in bone mineral density at the spine and hip. The authors concluded that the anti-sclerostin antibody was effective in the treatment of low bone mineral density in postmenopausal women.

     

    Illustration A is a comparison of the unsuppressed Wnt pathway (left) with the Wnt- antagonized pathways (right). Sclerostin binds the LRP5/6 receptor in the place of Wnt, leading to the release of the destruction complex and β-catenin degradation. In the absence of sclerostin, the β-catenin translates into the nucleus and promotes downstream transcription of Wnt target genes.

    Incorrect Answers:

    Answer 2: Sclerostin antagonizes Wnt, thereby INHIBITING osteoblastic differentiation.

    Answer 3: Sclerostin antagonizes Wnt thereby indirectly PROMOTING (by inhibiting the inhibition of) osteoblastic differentiation.

    Answer 4: Sclerostin antagonizes Wnt binding to the LRP5/6 receptor. Answer 5: SOST gene mutations resulting in decreased expression of

    sclerostin lead to an under- or uninhibited Wnt/β-catenin pathway and thereby INCREASED bone mass, such as would be seen in Van Buchem disease or sclerosteosis.









     

    Figures A through C are the radiographs and CT scan of a 33- year-old male who was treated 13-months ago for an open tibial shaft fracture. He has received several courses of intravenous antibiotics for chronic osteomyelitis. Despite continued treatment with IV antibiotics, his inflammatory markers remain elevated. The decision is made to proceed with irrigation and debridement, nail removal with exchange for a polymethylmethacrylate intramedullary nail with vancomycin and tobramycin. What is the primary mechanism of action of tobramycin?



    Anti-sclerostin antibodies result in increased bone density Sclerostin induces osteoblastic differentiation Sclerostin inhibits osteoblast and osteocyte apoptosis Sclerostin antagonizes BMP binding to the LRP5/6 receptor SOST gene mutations have been linked to hereditary osteoporosis
     

    Corrent answer: 2

    Lubricin is a hyaluronic acid-binding proteoglycan found in synovial fluid that reduces the coefficient of friction between the surfaces of the joint.

     

    Lubricin reduces the friction between the surfaces in the joint, leading to decreased shear forces transmitted to the hyaline cartilage. It is a glycoprotein that is produced by the chondrocytes in the superficial zone and is not a primary component of the extracellular matrix. A deficiency in lubricin has

    been associated with early-onset arthritis.

     

    Schumacher et al. first discovered what is now known to be "lubricin" by studying the superficial zone of bovine articular cartilage. The authors noted that the chondrocytes in this zone secreted this proteoglycan. In addition, they found that this molecule, or a very similar molecule, was present in synovial fluid and moreover could serve as a functional metabolic marker for chondrocytes of the superficial zone of articular cartilage.

     

    Jay et al. analyzed the synovial fluid in both normal and lubricin-deficient samples and found that the subdiffusive and elastic behavior of synovial fluid, at physiological shear rates, was absent in fluid from a patient who lacked lubricin. They concluded that lubricin provided synovial fluid with an ability to dissipate strain energy induced by physiologic motion, which is a chondroprotective feature distinct from boundary lubrication.

     

    Incorrect Answers:

    Answer 1: Decorin is the most abundant proteoglycan in tendons and is responsible for the regulation of collagen fiber diameter.

    Answer 3: Scleraxis is an important transcription factor for both tendon and ligament production and is also a negative regulator for bone formation by inhibiting

    osteoblastogenesis.

    Answer 4: Aggrecan is a proteoglycan that prevents swelling pressure against the restraint of collagen and helps distributes pressure evenly to articular cartilage.

    Answer 5: Cathepsin is a proteolytic enzyme produced by osteoclasts that digests organic matrix found at the ruffled order of bone.





     

    Which of the following is accurate regarding sclerostin?


    Decorin Lubricin Scleraxis Aggrecan Cathepsin
     

    Corrent answer: 1

     

    In the days following an intra-articular injury, the following substances are produced, contributing to articular cartilage damage and the eventual formation of post-traumatic arthritis: IL-1ß, TNF-a, nitric oxide, matrix metalloproteinases, aggrecans, and damage-associated molecular patterns.

     

    Immediately following an intra-articular fracture, there is mechanical damage and necrosis of articular cartilage. Traditionally, orthopaedic surgeons are

    taught that the most critical factor in affecting the outcomes of these patients is the accuracy of the articular reduction and restoration of the mechanical alignment. However, even in expertly reduced fractures, some patients experience poor outcomes and develop progressive, debilitating osteoarthritis. More recently, researchers have looked at inflammatory events that may also contribute to arthritis and ways to modulate these events.

     

    Olson et al. provide a review article on the role of cytokines in post-traumatic arthritis. They note that, despite accurate articular reductions, many patients go on to develop

    arthritic changes, often indistinguishable from primary OA. While mechanical alignment and structural damage are sometimes responsible, the cascade of cytokines and other signaling molecules listed above serve to catalyze these intra-articular events; developing ways to blunt this inflammatory response is of great interest.

     

    Lewis et al. examined the relationship of inflammatory and post-traumatic arthritis in a rodent model. Tibial plateau fractures were induced in C57BL/6 and MRL/MpJ "superhealer" mice, which were killed at different time-points. Synovial fluid was inspected post-mortem for cytokine analysis, as well as gross specimens, and it was determined that an association exists between joint tissue inflammation and the development and progression of post- traumatic arthritis in mice.

     

    Figure A is an XR of a tibial plateau fracture. Figure B is an XR of a knee demonstrating post-traumatic arthritis. Illustration A is a table of several cytokines and their functions. Illustration B is a timeline of intra-articular pathogenic events following an injury. Illustration C is a diagram showing various cellular events and pathogenic mechanisms in the acute aftermath following an intra-articular injury.

     

    Incorrect Answers:

    Answers 2, 3, 4, 5: FGF4 is attributed to bone formation, BMP6 is attributed to chondrogenesis, basic FGF is mitogenic, VEGF stimulates the formation of

    blood vessels, IL-8 is chemotactic, BMP2 plays a role in the development of bone and cartilage, BMP5 plays a role in cartilage development, and M-CSF causes hematopoietic stem cells to differentiate into macrophages or other related cells.








     

    A 45-year-old patient presents to your clinic for evaluation of knee pain. He has been told he has osteoarthritis and has significant pain with knee range of motion. Which of the following components of synovial fluid is most responsible for reducing the coefficient of friction in a native knee joint?



    IL-1ß, TNF-a, nitric oxide, matrix metalloproteinases, aggrecans, damage- associated molecular patterns FGF4, BMP6, nitric oxide, matrix metalloproteinases, basic FGF, VEGF IL-8, BMP2, TNF-a, damage-associated molecular patterns, aggrecans, BMP5 VEGF, M-CSF, IL-1ß, BMP2, BMP6, nitric oxide BMP5, basic FGF, nitric oxide, aggrecans, TNF-a, IL-1ß
     

    Corrent answer: 4

     

    This player has sustained a tear of the medial collateral ligament (MCL). The MCL is a ligament which inserts indirectly into bone through Sharpey's fibers.

     

    Ligaments can insert on bone either indirectly and directly. Indirect is the most common and is a fibrous insertion. The superficial fibers of the tendon insert into the periosteum, while the deep fibers insert directly into the bone. These

    deep fibers are called Sharpey's fibers and are made of type I collagen. The direct insertion has both deep and superficial fiber insertions as well. Direct insertions are fibrocartilaginous and consist of four transitional zones of increasing stiffness that allow force dissipation.

     

    Lu et al. performed a review to determine the functional attachments of soft tissue to bone. They report that a specialized interface, called an insertion site or enthesis, integrates

    tendon or ligament to bone and serves to facilitate joint motion. Fibrous (indirect) insertions typically occur over large areas, presumably to distribute force and reduce stress, and are characterized by perforating mineralized collagen fibers.

     

    Cole et al. performed a review of fixation of soft tissues to bone. They report that recreation of the enthesis relies on adequate biologic healing afforded by adequate initial fixation. The healing pattern associated with direct soft–tissue- to-bone repair, such as rotator cuff repair, is different from that associated

    with fixation within bone tunnels (ex. ACL reconstruction). The process of tendon healing within osseous tunnels occurs over time.

     

    Lui et al. performed a review of the biology and augmentation of tendon-bone insertion repair. They report that when a ligament runs parallel to the bone, like the MCL, the insertion is more likely to be indirect. When a ligament enters the bone perpendicularly, such as the ACL, the insertion is direct. Indirect insertions may be elevated off the bone without cutting the ligament itself while direct insertions require cutting the substance of the ligament to detach it.

     

    Figure A is a T2-weighted, coronal MRI demonstrating a tear of the MCL. Illustration A is a polarized photomicrograph demonstrating Sharpey fibers, indicated by the white arrows. G represents tendon, while B represents bone (Liu et al.). Illustration B is a Safranin-O-staining photomicrograph of a direct tendon insertion site (Liu et al.).

    Illustration C is an H&E photomicrograph of the same direct tendon insertion site (Liu et al.). B represents bone, CFC represents calcified fibrocartilage, UCFC represents uncalcified fibrocartilage, and T represents tendon.

     

    Incorrect Answers:

    Answers 1, 2, 3, & 5: The MCL is a ligament which inserts indirectly into bone through Sharpey's fibers.







     

    A 32-year-old male sustains the injury shown in Figure A. He undergoes surgical fixation with subsequent removal of hardware. He does well for 10 years and then presents to your office with increasing left knee stiffness and pain for the last 6 months. He reports no constitutional symptoms or recent trauma. His physical exam is notable for well-healed incisions, a mild effusion, no ligamentous instability, and 5-100 degrees of range of motion. An XR is obtained and shown in Figure B. Which of the following correctly lists the cytokines produced following the initial injury that may contribute to the findings shown in Figure B and the patient's current symptoms?

    imaging is shown in Figure A. Which of the following correctly classifies the injured structure and its indirect insertion into bone?





    Tendon; Sharpey's fibers Tendon; Fibrocartilagenous transitional zone Ligament; Fibrocartilagenous transitional zone Ligament; Sharpey's fibers None of the above
     

    Corrent answer: 1

     

    Hypertrophic nonunions are described as having abundant callous formation without bony bridging at the fracture site and rarely require an increase in bone biology to achieve fracture healing.

    A nonunion is defined as a fracture that has not healed and has no further capacity to heal without further intervention. Nonunions are typically classified as hypertrophic, oligotrophic, and atrophic. Hypertrophic nonunions show clear

    evidence of ability to heal without bridging of fracture gaps. Atrophic nonunions show no evidence of biologic healing and no bridging of fracture gaps. Oligotrophic nonunions tend to fall somewhere in between hypertrophic and atrophic nonunions with some evidence of biologic activity however incomplete healing. Understanding these characteristics allows for proper identification of the nonunion and selection of appropriate intervention with regard to increasing bone biology and fracture stability to achieve healing.

     

    Bishop et al. review the diagnosis and assessment of delayed bone healing through a systematic approach to help surgeons determine appropriate interventions to achieve healing. They state biologic capacity, fracture stability, deformity, infection, and host status should all be considered closely prior to establishing a plan of management for a nonunion.

     

    Babhulkar et al. reviewed 113 patients diagnosed and treated for nonunions including 61 hypertrophic and 52 atrophic nonunions. They found all patients healed with improved function and pain following treatment of their nonunions. They found residual problems related to joint stiffness, limb length

    discrepancy, and angular deformity.

     

    Illustration A shows a hypertrophic nonunion of a tibial shaft fracture treated nonoperatively. Illustration B shows an atrophic nonunion following open reduction internal fixation of a humeral shaft fracture.

     

    Incorrect Answers:

    Answer 2: Oligotrophic nonunions are characterized by some evidence of preserved biology but the inability to completely heal the fracture. They are often treated by altering the mechanical stability and often need alteration to bone biology.

    Answer 3: Atrophic nonunions are characterized by no callous formation with sclerotic bone edges at the fracture site and almost always require alterations to bone biology. Answer 4: A bone defect filled with a cement spacer describes the Masquelet technique. The cement spacer is removed and the void is often filled with some form of bone graft after approximately 6 weeks.

    Answer 5: This describes an infected nonunion. Treatment of infected nonunions is complex and often requires staged procedures of debridement, eradication, and stabilization.



     

    A 20-year-old male collegiate football player sustained a knee injury. His


    Abundant callous formation without bony bridging at the fracture site Small callous formation without bony bridging at the fracture site No callous formation with sclerotic bone edges at the fracture site A segmental bone defect that was filled with an antibiotic cement spacer 6 weeks prior Fracture site contamination with elevated inflammatory markers
     

    Corrent answer: 3

     

    The arrow is pointing to the superficial zone of articular cartilage in Figure A. The superficial zone of articular cartilage has the highest content of collagen and the lowest content of proteoglycans relative to the other zones.

    Normal articular cartilage can be divided into 3 zones and the tidemark based on the shape of the chondrocytes and the orientation of the type II collagen. The zones, in order from closest to the joint surface, are superficial zone, intermediate (transitional) zone, and deep zone. The superficial zone has the highest content of collagen and lowest content of proteoglycans of all the zones; in contrast, the deep zone has the lowest content of collagen and the highest content of proteoglycans. The intermediate zone has amounts of collagen and proteoglycans that reside between those found in the superficial and deep zones.

     

    Ulrich-Vinther et al. reviewed the biology of articular cartilage. They noted three distinct zones of articular cartilage that are separated from the subchondral bone by the tidemark. The authors noted that tissue engineering approaches are being used in an effort to regenerate damaged articular cartilage due to injury or aging.

     

    Jeffery et al. studied the three-dimensional architecture of bovine articular cartilage with scanning electron microscopy. They noted that the collagen was ordered in different morphologies in each zone of articular cartilage. The authors concluded that the three- dimensional organization of collagen is important when considering cartilage structure and function.

     

    Illustration A demonstrates the zones of articular cartilage and their spacial relationship. Illustration B shows the relationship between collagen and proteoglycans

    within articular cartilage.

     

    Incorrect Answers:

    Answers 1 and 2: These choices do not describe any zones of articular cartilage Answer 4: This description correctly describes the deep zone of articular cartilage Answer 5: This description correctly describes the intermediate (transitional) zone of articular cartilage






     

    While planning for revision of a failed open reduction internal fixation you are planning to increase mechanical stability across the fracture site. In addition to addressing stability, which of the following fracture scenarios is least likely to require additional bone biology in order to achieve healing?

    A is characterized by:





    High collagen and proteoglycan content Low collagen and proteoglycan content High collagen and low proteoglycan content Low collagen and high proteoglycan content Intermediate amounts of both collagen and proteoglycan

    Corrent answer: 1

    PROMIS is a useful orthopaedic patient-reported outcomes measure (PROM) that can assess disease specific and general health questions, validated for use in foot and ankle, upper extremity, and spine patients.

    Multiple tools have been developed to asses patient-reported outcomes, however, some of these are narrow in scope, cumbersome to administer, and less useful in orthopedics. PROMIS was developed by a team of NIH researchers to address these shortcomings. Domains include physical, mental, and social health. PROMIS utilizes computer adaptive testing software (CAT), allowing for fewer questions and more accurate measurements. Additionally, results are reported as T-scores, allowing for easy interpretation.

    Brodke et al. provide an overview of PROMIS. They report that this tool was developed to be easy to administer and broad in scope, utilizing item response theory. This validated tool is reliable in assessing specific function of the upper and lower extremities, as well as underlying health traits.

     

    McCormick et al. described PROM used in spine surgery. They reinforce that the benefit of PROM is to gather insight into subjective measures not typically captured in traditional research, looking at such factors as general health quality, function, and pain. The authors also note the importance of these tools as greater emphasis is placed on quality of care and patient experience.

     

    Godil et al. studied instruments to accurately measure quality and outcomes in lumber spine surgical registries. They conducted a prospective cohort study of

    58 patients undergoing TLIF for lumbar degenerative spondylolisthesis and administered several PROs. They concluded that the ODI was the best measure assessing pain and disability in lumbar surgery, citing its validity and responsiveness in measuring the effectiveness of lumbar fusion; EuroQOL-5D (EQ-5D) was found to be the best, the most valid, and responsive measure of improvement for health-related quality of life.

     

    Illustration A is a chart depicting the main domains and subcategories analyzed with PROMIS.

     

    Incorrect Answers:

    Answer 2: SRS-22 is one of the most commonly used measures of general health. Answer 3: ODI measures disability in patients with back pain due to multiple causes. Answer 4: SF-12 assesses general physical and mental health.

    Answer 5: PHQ-9 is a tool to screen for both the presence and severity of depression.





     

    The layer of articular cartilage that the arrow points to in Figure


    Patient-Reported Outcomes Measurement Information System (PROMIS) Scoliosis Research Society-22 (SRS-22) Oswestry Disability Index (ODI) Short Form 12-Item (SF-12) Patient Health Questionnaire-9 (PHQ-9)

    Corrent answer: 3

     

    As part of the World Health Organization (WHO) Surgical Safety Checklist, all of the answers listed are methods to prevent wrong-site surgery EXCEPT marking an "X" on the operative extremity. Patients should be marked unambiguously, with either a "yes" or the surgeon's initials in permanent marker, prior to induction of anesthesia.

     

    The WHO developed the Surgical Safety Checklist in an effort to improve the safety of patients undergoing surgery. Implementation of this checklist has resulted in improved clinician safety attitudes, as well as decreased patient morbidity and mortality. A time-out or group huddle occurs prior to induction with the patient, prior to incision, and prior to the patient leaving the operating room; all team members have an opportunity to speak up and discuss any concerns during this process. The American Academy of Orthopaedic Surgeons suggest the following to prevent wrong-site surgery: Surgical team engagement, patient confirmation, signing the surgical site (in the visible

    surgical field or inline with the planned incision) with a permanent marker with the patient's assistance, and utilizing separate time-outs in the case of separate surgical procedures/sites.

     

    Haynes et al. looked at changes in safety attitude, morbidity, and mortality following implementation of the WHO Surgical Safety Checklist. They administered a survey pre- and post-intervention at 8 hospitals. Post- intervention, they found an overall improvement in safety attitudes and found that this was correlated with a reduction in post-operative complication rates.

     

    Gillespie at al. reviewed the evidence of implementing a surgical safety checklist. They utilized a realist synthesis methodology in this study. They concluded that intervention methods and implementation strategies were not well described in the literature, surgical checklists appear to be more successful when physicians are leading their implementation, and that greater participation and ownership of safety checklists can be expected by physicians are actively engaged in their development and implementation.

     

    Illustration A is the WHO Surgical Safety Checklist. Illustration B is an example of the correct way to mark a patient for a right shoulder surgery for a planned deltopectoral incision.

     

    Incorrect Answers:

    Answers 1, 2, 4, and 5 are components of the WHO Surgical Safety Checklist designed to increase patient safety and eliminate the risk of wrong site surgery.








     

    Which of the following instruments incorporates both general disease and disease-specific measures and has been validated for use in patients with spine, foot and ankle, and upper extremity conditions?


    Performing a sign-in/huddle with the patient and surgical staff Displaying pertinent imaging in the operating room Marking an "X" at the operative site directly over the planned incision Marking the surgeon's initials at the operative site within the visible surgical field Performing a time-out with the surgeon, anesthesia professional, and nurse prior to incision
     

    Corrent answer: 5

     

    This patient has a catastrophic ceramic component failure and requires a thorough debridement and revision with a head and liner exchange.

    Ceramic bearings in hip arthroplasty are recognized for their superior wear properties and low-friction. However, ceramics are also brittle, have a high modulus of elasticity, and are prone to fracture under certain circumstances. Catastrophic bearing failure is not as common in newer-generation ceramics, as first-generation products were more prone to failure due to flaws in the manufacturing process. Squeaking is a known complication of ceramic-on- ceramic bearings and may be associated with catastrophic failure. Obesity, trauma, and component malposition have been linked to failure, and revision procedures should address any component malposition.

     

    Malem et al. describe a case report of a catastrophic ceramic-on-ceramic total hip replacement failure presenting as a squeaking hip. Within 5 years of her index surgery, the patient developed a painful, squeaking hip with a limited range of motion. At the time of revision, she was found to have a broken femoral head, black wear debris, and a completely worn acetabular component, suggesting that a squeaking ceramic-on-ceramic hip replacement may be a sign of catastrophic failure.

    Stanat et al. provide a meta-analysis and review of squeaking in 3rd and 4th generation ceramic-on-ceramic total hip replacements. They conclude that the only significant patient- associated risk factor for squeaking was body mass index. In terms of implant type and surgical factors, they found that the presence of a Stryker Accolade stem was associated with an increased incidence of squeaking; cup version was not associated with a higher incidence of squeaking in their analysis.

     

    Figure A is a radiograph demonstrating ceramic fracture and displacement of the femoral component in relation to the acetabular component; ceramic fragments are present around the femoral neck and cup. The partially radiolucent head is a tip-off to the older generation ceramics. Illustration A is an intra-operative photograph demonstrating a fractured ceramic femoral head. Illustration B is a revision of the hip in Figure A utilizing ceramic-on- ceramic components. Illustration C (Traina et al.) is a treatment algorithm when ceramic bearing failure is suspected.

     

    Incorrect Answers:

    Answers 1 and 2: Observation, physiotherapy, and corticosteroid injections are not indicated in the setting of catastrophic implant failure.

    Answer 3: Left hip aspiration is not necessary in this example, as the patient has clear evidence of fractured ceramic components, a well-healed incision, and normal inflammatory markers; frozen samples and cultures can be obtained at the time of revision if desired.

    Answer 4: As there is no reason to suspect a prosthetic joint infection, one can proceed with a one-stage revision in this setting.











     

    All of the following can be done to improve patient safety and prevent wrong site surgery EXCEPT:



    Observation and physical therapy Left hip corticosteroid injection Left hip aspiration Thorough debridement and two-stage revision Thorough debridement and one-stage head and liner revision
     

    Corrent answer: 1

    Nitrogen-containing bisphosphonates exert their action by inhibiting farnesyl pyrophosphate synthase.

     

    Nitrogen-containing bisphosphonates inhibit osteoclasts which leads to increased bone mass and reduced bone turnover. They act on the cholesterol biosynthesis pathway enzyme, farnesyl pyrophosphate synthase. By inhibiting this enzyme in the osteoclast, they interfere with geranylgeranylation (attachment of the lipid to regulatory proteins), which causes osteoclast inactivation. Non-nitrogen containing bisphosphonates are metabolized in the cell to compounds that replace the terminal pyrophosphate moiety of ATP, forming a nonfunctional molecule that competes with ATP in the cellular energy metabolism. Due to this disruption in metabolism, the osteoclast

    initiates apoptosis and dies, leading to an overall decrease in the breakdown of bone. Examples of nitrogen-containing bisphosphonates are alendronate, risedronate, pamidronate, and zolendronate. Examples of non-nitrogen containing bisphosphonates are tiludronate, clodronate, etidronate.

     

    Reszka et al. performed a review of bisphosphonates. They report that they bind to the bone mineral, which localizes their action to the target tissue. They are rapidly cleared from the circulation via renal excretion, which minimizes exposure of all other organs.

    They do not easily penetrate cell membranes, which reduces exposure to non-target tissues. They also report that alendronate and risedronate are the only pharmacologic agents shown to prevent spine and nonvertebral fractures associated with postmenopausal and glucocorticoid-induced osteoporosis.

     

    Luo et al. performed a systematic review to determine the efficacy and safety of alendronate for adult AVN treatment. They found most studies suggested a positive short-term efficacy of alendronate treatment in reducing pain, improving articular function, slowing of bone collapse progression, and delaying the need for arthroplasty. There were no severe adverse effects associated with alendronate treatment observed and most of the included studies suggested the use of alendronate in early AVN with a small necrotic lesion to achieve better outcomes.

     

    Illustration A (Reszka et al.) details the nitrogen and non-nitrogen containing bisphosphonates and their chemical structures. Illustration B is the mevalonate pathway, showing the action of a nitrogen-containing bisphosphonate.

     

    Incorrect Answers:

    Answer 2: Non-nitrogen bisphosphonates form a molecule that competes with ATP in the cellular energy metabolism.

    Answer 3: Selective estrogen receptor modulators, such as raloxifene and bazedoxifene, have estrogen activity in bone and, therefore, prevent bone loss, improve bone mineral density, and decrease the risk of vertebral fracture.

    Answer 4: Corticosteroids are used to treat many pathologic conditions. One of its possible side effects is osteonecrosis.

    Answer 5: This answer choice is referring to teriparatide. Teriparatide is an analog of parathyroid hormone and activates osteoblasts.







     

    A 60-year-old patient presents to clinic complaining of left hip squeaking and stiffness following a ceramic-on-ceramic total hip replacement five years earlier. Initially, he was extremely pleased with the operation, but these symptoms of squeaking and stiffness have developed fairly abruptly over the past 6 months. On physical exam, he has a well-healed incision, a 1 centimeter leg length discrepancy (left shorter than right), decreased hip flexion, and decreased internal rotation. A radiograph is shown in Figure A. C- reactive protein is 0.2 mg/dL (nl <0.9 mg/dL) and erythrocyte sedimentation rate is 5 mm/hr (nl 0-15 mm/hr). What would you recommend for this patient?

    in the red box?





    Nitrogen-containing bisphosphonate Non-nitrogen containing bisphosphonate Selective estrogen receptor modulators Corticosteriods Recombinant protein form of parathyroid hormone
     

    Corrent answer: 3

     

    Aspirin inhibits the aggregation of platelets, which are shown in Figure C.

     

    The coagulation cascade is a complex interaction of multiple clotting factors which ultimately leads to the aggregation of platelets to form a clot. The end result of the clotting cascade leads to the conversion of prothrombin to thrombin, causing the formation of fibrin and the subsequent aggregation of platelets in a clot. Aspirin irreversibly blocks the formation of thromboxane A2 in platelets, which inhibits platelet aggregation and thus the formation of a clot.

     

    Hyers reviewed the mechanism of action of various anticoagulants and the pathophysiology of venous thromboembolism. He noted that while deep vein thrombosis and pulmonary embolism resulted in significant morbidity and mortality in the United States, novel anticoagulants have recently been developed that hold promise for new therapeutic options.

     

    Brown pooled the results of 14 randomized controlled trials examining venous thromboembolism (VTE) rates. Their analysis showed that VTE rates were not significantly different when aspirin was used for anticoagulation compared with vitamin K antagonists or low molecular weight heparin (LMWH). The authors concluded that aspirin could be used for VTE prophylaxis after major orthopaedic surgery.

     

    Figure A shows a plasma cell. Figure B shows a neutrophil. Figure C shows arrows pointing to platelets in a peripheral blood smear. Figure D features a basophil, and figure E shows an osteoclast. Illustration A shows the mechanism of action of aspirin.

     

    Incorrect Answers:

    Answers 1, 2, 4, and 5: Aspirin does not work to prevent clotting through plasma cells, neutrophils, basophils, or osteoclasts.

     

    A man with osteonecrosis of the hip is treated with a medication which inhibits the pathway shown in Figure A. Which of the following medications best matches this mechanism of action of the medication









    A B C D E
     

    Corrent answer: 2

     

    Demineralized bone matrix (DBM) is both osteoconductive and osteoinductive.

     

    DBM is made from the acidic extraction of bone matrix from allograft. It removes the minerals and leaves the collagenous and noncollagenous structure and proteins. It is osteoconductive and osteoinductive as it contains collagen, bone morphogenic proteins (BMPs), transforming growth factor-beta (TGF-β), and residual calcium. It does not contain mesenchymal precursor cells or impart structural support.

     

    Ripamonti et al. performed a review of regenerative medicine and tissue engineering of bone. They report that the understanding of bone formation by autoinduction has been pivotal for setting the rules for tissue engineering. An extracellular matrix combined with TGF-β is capable of performing this task.

     

    Grabowski et al. review bone graft and bone graft substitutes. They report that DBM is both osteoinductive and osteoconductive. DBM provides varying degrees of osteoconductive potential based on the carrier material chosen.

    They also report that given the techniques used for demineralization are proprietary, the actual techniques are not published, and the process is not regulated. They conclude that the actual concentration of BMPs is varied when comparing various lots of the same preparation.

     

    Figure A is a coronal CT slice demonstrating a depression fracture of the lateral tibial plateau.

     

    Incorrect Answers:

    Answer 1: Calcium phosphate cement is osteoconductive only. Answer 3: Bone morphogenic protein is osteoinductive only.

    Answers 4 & 5: Both cortical and cancellous allograft are osteoconductive only due to a lack of viable cells and growth factors. There is also no extracellular matrix present.





     

    Aspirin exerts its primary effect on which of the following cells (identified by black arrows)?





    Calcium phosphate Dimineralized bone matrix Bone morphogenic protein Cancellous allograft Cortical allograft

     

    FOR ALL MCQS CLICK THE LINK ORTHO MCQ BANK